Hit Counter

Authorized Blogger

Latest Posts

Saturday, February 12, 2011

National Current Affairs 2011

National Current Affairs 2011

Karnataka Governor sanctions prosecution of CM Yeddyurappa
Karnataka Governor HR Bharadwaj on 21/1/2011 sanctioned the prosecution of Chief Minister Yeddyurappa on charges of corruption. This is for the first time in Karnataka’s political history that a governor has permitted the prosecution of a chief minister. The Governor gave sanction for prosecution of the Chief Minister who is said to be involved in alleged land scams on a petition made by two lawyers. With this sanction, the lawyers will now approach a local court and will seek a court directive to the police for registering an FIR against the Chief Minister Yeddyurappa and the State Home Minister R Ashoka.


Officer burnt alive by Oil Mafia
Yashwant Sonawane, Additional District Collector of Malegaon (Maharashtra) was tragically burnt alive by oil mafia at Manmad near Nashik on 25 January 2011. Mr. Sonawane was returning from Chandrar village after reviewing the situation in the wake of a demonstration by onion farmers, when he found some people adulterating kerosene in a tanker parked behind Sagar Dhaba at Panewadi near Manmad. The area has a notorious reputation for smuggling and adulteration of oil. Sonawane stopped and questioned them and also shot a video of them pilfering the kerosene on his mobile phone. The main accused Popat Shinde and three others allegedly poured kerosene on the official, and set him afire. Mr. Sonawane died on the spot. The horrifying incident has caused a lot of shock and anger all across India.


Gujarat attracts Rs 20.83 lakh Crore investment in 2 days
Leading industrialists of India have flocked to Gujrat to sign MOU’s during the two-day long Vibrant Gujarat Summit that began on 1/12/2011. Chief Minister announced that the state has been able to attract investments to the tune of Rs 20.83 lakh crores during the two-day long Summit. As many as 7,936 MOUs were signed by the state government with several industrial houses to invest in the state. It is rare for 60 of India’s top Industrialists representing companies that command the bulk of market capitalization, to share the dais at a single event and it is even rarer to make investments to the tune of thousands of crores for a single state and that too over just two days. The major investments include Anil Ambani group, Adani Group, Essar Group, Larsen & Toubro, Mahindra & Mahindra and many others.


Australian firm set to sue Delhi CWG over non payment of dues.
In yet another case of embarrassment for India an Australian firm is set to sue the CWG organizers regarding non payment of bills. It is reported that the firm had played a very key and major role in the opening and closing ceremonies of the million dollar sporting event. Sydney Olympics maestro Ric Birch, who was creative director for the Delhi ceremonies, is likely to launch a multi-million dollar class action against Commonwealth Games organizers for substantial unpaid bills. The Australian government has already raised the issue with External Affairs Minister SM Krishna who has promised to look into the matter.
102 lives lost in Sabarimala Stampede


Around 102 pilgrims were killed and another 100 injured in a stampede about 40-45 km from the Sabarimala Shrine. The stampede was triggered when a jeep and an auto rickshaw rammed into a crowd of devotees in a narrow forest path at Pulimedu in Idukki district of Kerala on 14/1/2011. Most of the victims are from the Southern state of Tamil Nadu, Karnataka and Andhra Pradesh. The pilgrims were returning after seeking the blessings at the Sabarimala temple on the auspicious day of Makar Sankranti. The temple attracts millions of worshippers and an estimated 150,000 devotees were thought to have taken the narrow path out of the densely forested hills where the stampede took place. CM V.S Achuthanandan has launched a judicial probe into the matter.
Suresh Kalmadi sacked as Chairman of Commonwealth Games Organizing Committee
Suresh Kalmadi and his deputy Lalit Bhanot were sacked as Chairman and Secretary General of the Commonwealth Games Organizing Committee respectively after three months since the end of the international event. The decision to remove the two top officials was taken by the new Sports Minister Ajay Maken after taking the legal opinion of the Attorney General. The organizing committee has faced several allegations of massive corruption and ineptitude in the months leading up to the games.
Aarushi Talwar's father attacked outside court
Dr. Rajesh Talwar father of murdered teenager Aarushi Talwar was attacked by a man outside the Ghaziabad court on 25/1/2011. He received sharp blows from a meat cleaver used by the assailant and the bleeding doctor was immediately rushed to a Hospital. Rajesh Talwar had gone to the court to file a petition demanding a fresh probe by the CBI in the Aarushi- Hemaraj murder case. The attacker has been identified as Utsav Sharma who was earlier also involved in an incident where he attacked SPS Rathore, the retired police officer who was convicted in the Ruchita Girhotra suicide case.


Republic Day celebrations peaceful in Kashmir
India’s Republic Day celebrations in Kashmir passed of peacefully even as BJP activist’s attempts to hoist the flag at Lal Chowk were foiled. Six activists were taken into custody immediately after they came out of a hotel near Lal Chowk and began walking towards the city centre. Another activist was arrested in Residency Road area adjacent to Lal Chowk when he appeared with a national flag and the BJP flag. The days leading to the Republic Day were fraught with tension as BJP’s proposed ‘EktaYatra’ was seen as a move that could possibly reignite tensions in the valley.
read more...

Thursday, August 5, 2010

Changing Pattern Of UPSC Examination

The Government has approved the proposal for introduction of Civil Services Aptitude Test (CSAT) in place of Civil Services (Preliminary) Examination. The CSAT is expected to come into effect from Civil Services Examination, 2011 would be transparent. This will enable screening of candidates having a right aptitude for Civil Services.

In CSAT, one of optional subjects, which a candidate could have chosen out of 23 optionals, is being replaced with a common paper on aptitude test. The syllabus of CSAT is being worked out by the UPSC. The CSAT is aimed at providing a greater degree of level - playing field to candidates of different backgrounds.

This information was given by the Minister of State in the Ministry of Personnel, Public Grievances & Pensions, Shri Prithviraj Chavan in written reply to a question in Lok Sabha today.

Courtesy: Press Information Bureau (PIB)
Wednesday, August 04, 2010
read more...

Tuesday, May 25, 2010

UP PCS (Pre) 2010 GENERAL STUDIES

1. Dhuria is folkd dance of
(a) Avadh
(b) Bundelkhand
(c) Purvanchal
(d) Rohelkhand
Ans. b
Explanation: Dhuria is the dance form of tribal. This dance form is very famous in Bundelkhand region

2. In which one of the following States is Suil river project located?
(a) Uttarakhand
(b) Haryana
(c) Punjab
(d) Himachal Pradesh
Ans. d
Explanation:
Suil river project is located in Himachal Pradesh.

3. Consider the following statements:
Assertion (A) : River Damodar was known as "River of Sorrow" in west Bengal prior to the development of the Damodar Valley Corporation.
Reason (R) : Damodar in its upper reaches flows rapidly and in its lower reaches it runs too sluggishly.
Select the correct answer using the codes given below:
Codes
(a) Both (A) and (R) are true and (R) is the correct explanation of (A).
(b) Both (A) and (R) are true, but (R) is not the correct explanation of (A).
(c) (A) is true, but (R) is false.
(d) (A) is false, but (R) is true.
Ans. a
Explanation:
Damodar River originates near Chandwa village, Palamau district, on the Chota Nagpur Plateau in the Jharkhand. The Chota Nagpur Plateau receives an average annual rainfall of around 1400 mm, almost all of it in the monsoon months between June and August. The huge volume of water that flows down the Damodar and its tributaries during the monsoons used to be a fury in the upper reaches of the valley but in the lower valley it used to overflow its banks and flood large areas.

4. Which State has decided to establish a University for the disabled during 2009-10?
(a) Maharashtra
(b) Kerala
(c) Madhya Pradesh
(d) Uttar Pradesh
Ans. d
Explanation: The U.P. Jagadguru Rambhadracharya Handicapped University was established to provide higher & professional education to persons with disabilities.

5. Which one of the following is not included in the "National Food Security Missions"?
(a) Oil seeds
(b) Wheat
(c) Rice
(d) Pulses
Ans. a
Explanation: The National Development Council (NDC)in its 53rd meeting held on 29th May, 2007 adopted a resolution to launch a Food Security Mission comprising rice, wheat and pulses to increase the production of rice by 10 million tons, wheat by 8 million tons and pulses by 2 million tons by the end of the Eleventh Plan (2011-12).

6. March List-I with List-II and select the correct answer from the codes given below the lists.
List-I List-II
A. Kishan Maharag 1. Sarangi player
B. Hari Prasad 2. Tabla
player
C. Pt. Gopalji 3. Pakhawaj
Mishra player
D. Kudak Singh 4. Bansuri
player
Codes:
A B C D
(a) 1 3 4 2
(b) 3 4 2 1
(c) 2 4 3 1
(d) 2 4 1 3
Ans. d
Explanation:
Hari Prasad- Bansuri player
Kishan Maharag -an Indian tabla player who belonged to the Benares gharana.
Pt. Gopalji Mishra- Sarangi player

7. Which one of the following is not correctly matched?
(a) Alha - Bundelkhand
(b) Birha - Purvanchal
(c) Chaiti- Rohelkhand
(d) Kajri - Avadh
Ans. c
Explanation:
Chaiti is sung in the month of Chait that falls in March/April as per the Hindu calendar. This is certainly a season specific song. This season has a great importance in the largely agrarian society of Eastern Uttar Pradesh and Bihar for this is the season of harvest.
Birha is extremely popular among the farmers and laborers in eastern Uttar Pradesh and Bihar.
Even though Kajri is sung in a large region - Mirzapur is considered the real home of the Kajri.

8. Which one of the following is not a fold dance of Uttar Pradesh?
(a) Charkula
(b) Dadra
(c) Karma
(d) Muria
Ans. d
Explanation:
The dances in Uttar Pradesh can be broadly classified in to two categories namely Classical and Folk. The first group comprises Kathak where as the later has Charkula, Karma and Dadra in its list.

9. Sukna scam case was recetly (first quarter of 2010) much in the news. With which State is it concerned?
(a) Himachal Pradesh
(b) Jammu and Kashmir
(c) Rajasthan
(d) West Bengal
Ans. d
Explanation:
Sukhna land scam case is concerned with West Bengal.The Court of Inquiry into the Sukhna land scam case, in which three other Army generals have been indicted, holds Lt General Prakash responsible for allegedly hiding information and taking undue interest in promoting the vested agenda of real estate developer and family friend Dilip Agarwal.

10. The author of the book which recently became the root cause of the controversy regarding the script of the film "Three Indiots" is
(a) Adhijat Joshi
(b) Mohammad Khalid
(c) Chetan Bhagat
(d) Rajkumar Hirani
Ans. c

11. Consider the following statements:
1. The crop insurance scheme in India was started in the year 1985.
2. The total number of agroclimatic zone in U.P. is 9.
3. Food for work programme was started in the year 1977.
4. Blue resolution is concerned with production of mustard.
Of these statements:
(a) only 1 and 2 are correct
(b) only 2 and 3 are correct
(c) only 3 and 4 are correct
(d) only 1, 2 and 3 are correct
Ans. d
Explanation:
Since Independence, India has witnessed
significant increase in foodgrain production (green revolution), oilseeds (yellow revolution), milk (white revolution), fish (blue revolution), and fruits and vegetables (golden revolution).

12. In which year the Government of India included ten new tribes of Uttar Pradesh as Scheduled Tribes?
(a) 2004 (b) 2003
(c) 2002 (d) 2001
Ans. b
Explanation:
In 2003, the Government of India included ten new tribes of Uttar Pradesh as Scheduled Tribes.

13. In Uttar Pradesh, which one of the following districts has the largest population of Scheduled Tribes?
(a) Sonebhadra
(b) Mirzapur
(c) Kheri
(d) Bijnore
Ans. c
Explanation: District wise distribution of ST population shows that Kheri district has the highest proportion of STs (1.2 per cent), followed by Balrampur (1.1 per
cent).

14. How many nations were members of the BASIC Block at the December 2009 Copenhangen Meet?
(a) Five (b) Four
(c) Three (d) Two
Ans. b
Explanation: The BASIC countries (also Basic countries, BASIC or G4) are a bloc of four large developing countries – Brazil, South Africa, India and China – formed by an agreement on 28 November 2009. The four committed to act jointly at the Copenhagen climate summit, including a possible united walk-out if their common minimum position was not met by the developed nations.

15. The Rangnath Mishra Commission report which was tabled in the Parliament in December 2009, relates to
(a) Electoral reforms
(b) Police reforms
(c) Religious and Linguistic Minorities
(d) Tax reforms
Ans. c
Explanation: Ranganath Mishra Commission is an enquiry commission assigned by Government of India to study and find solution for the minority status of India. The report was conducted by National Commission for Religious and Linguistic Minorities.
Main Findings
*15% of jobs in government services and seats in educational institutions for minorities
*reserves 8.4% out of existing OBC quota of 27% for minorities
*SC reservation to Dalit converts[1]

16. Match List-I with List-II and select the correct answer using the codes given below the lists.
List-I List-II
(Name of (The writing
Film) inspiring the
film)
A. 3 Idiods 1. Pygmalion
B. Slum dog 2. Five Point some
Millionaire one
C. Junoon 3. Q and A
d. My Fair Lady 4. A Flight Pigeons

Codes:
A B C D
(a) 2 1 4 3
(b) 3 4 1 2
(c) 3 1 4 2
(d) 2 3 4 1
Ans. d

17. Who has written the book "The Rediscovery of India" published around November 2009?
(a) Ram Chandra Guha
(b) Meghnad Desai
(c) Arun Shourie
(d) Mohan Bhagwat
Ans. b
Explanation: book "The Rediscovery of India"- Meghnad Desai

18. Which country lunched the World's first satellite dedicated to monitoring "greenhouse gas emissions" in 2009?
(a) Japan (b) Brazil
(c) India (d) U.S.A.
Ans. a
Explanation: The first satellite dedicated to monitoring carbon dioxide emissions was launched into space from a center in Japan. The satellite -- named "Ibuki," which means "breath" -- was sent into orbit along with seven other piggyback probes on a Japanese H2A rocket. Japan's space agency-JAXA

19. "Project Arrow" is concerned with the modernisation of which of the following?
(a) Airports
(b) Post offices
(c) Road Transport
(d) Railways
Ans. b
Explanation: The department of posts (DoP) launched ‘Project Arrow’, an initiative to transform India Post into a vibrant and responsive organization and to make a visible and positive difference in postal operations to benefit the customers, informed a senior department official.

20. During Prime Minister Man Mohan Singh's visit to Russia in December 2009, India and Russia signed an agreement mainly relating to
(a) Civil nuclear cooperation
(b) Climate change
(c) Cooperation in agriculture sector
(d) Cooperation in science and technology sector
Ans. a
Explanation: Indian Prime Minister Manmohan Singh inked six major defense and nuclear deals including a crucial agreement on the use and cooperation of atomic energy during his three-day Russia visit.

21. The city which has been selected by a famous International magazine "Travel and Leisure" in its survey 2009 as the best city from tourism point of view in the world is
(a) Udaipur (b) Hongkong
(c) Singapore (d) Dubai
Ans. a
Explanation: Udaipur has been named the World's Best City in a poll by Travel and Leisure magazine. The Rajasthani city is the first South Asian hotspot to get the prestigious title. New York has been voted the 'Best City in America', for the ninth year in a row.
22. Football World Cup 2010 will be held in
(a) Britain (b) Germany
(c) Portugal (d) South Africa
Ans. d
Explanation: The 2010 FIFA World Cup will be the 19th FIFA World Cup, the premier international football tournament. It is scheduled to take place between 11 June and 11 July 2010 in South Africa. This will be the first time that the tournament has been hosted by an African nation, after South Africa beat Morocco and Egypt in an all-African bidding process.

23. How many seats BSP won in the elections to the U.P. Legislative Council held in January 2010?
(a) 31 (b) 32
(c) 33 (d) 34
Ans. d
Explanation: BSP has swept Legislative Council elections held for 36 seats, winning 33 seats.

24. The Indian citizen who has been honoured with Ramon Magasaysay award for the year 2009 is
(a) Bhimsen Joshi
(b) Deep Joshi
(c) Indira Sinha
(d) Pankaj Srivastava
Ans. b
Explanation: Deep Joshi is an Indian social worker and NGO activist and the recipient of 2009 Magsaysay awards announced. He was recognised for his vision and leadership in bringing professionalism to the NGO movement in India. He co-founded a non-profit organisation, Professional Assistance for Development Action (PRADAN) of which he is the Executive Director. He was awarded the 2009 Magsaysay award for Community Leadership for his work for "development of rural communities".

25. At the 56th National Film Awards for 2008 the best feature film award was bagged by
(a) Antaheen (b) Fashion
(c) Jogeva (d) Rock on
Ans. a
Explanation: BEST FEATURE FILM: Antaheen (Bengali)-Director : Aniruddha Roy Chowdhury; “For lyrical blend of technical devices in the right proportion to depict shifting human relationships in an urban scenario”.

26. Commonwealth Games are scheduled to take place in New Delhi, India in 2010. Which location has been chosen for the next Commonwealth Games in 2014?
(a)Brisbane - Australia
(b) Victorai - Canada
(c) Auckland - New Zealand
(d) Glasgow - Scotland
Ans. d
Explanation: The 20th Commonwealth Games in 2014 will be held in Glasgow, the largest city in Scotland. It will be the largest multi-sport event ever held in Scotland, although the country previously hosted the Games in 1970 and 1986 in Edinburgh.

27. Suresh Kalmadi has been elected Chairman of the 'Asian Athletic Association' on November 9th 2009 for the
(a) 2nd term (b) 3srd term
(c) 4th term (d) 5th term
Ans.
Explanation: The Asian Athletics Association is the Asian governing body for the sport of athletics. It is headquartered in Singapore. It organises the Asian Championships in Athletics and other continental competitions.Suresh Kalmadi of India is the current president. He was first elected to the position in 2000 and was re-elected for a third time in 2009.

28. ICC announced a list of 55 Cricket players in 2009 to be included in its inaugural "Hall of Fame" list.
Identify from the following cricketers who was not included in the list.
(a) Kapil Dev
(b) Sachin Tendulkar
(c) Sunil Gavaskar
(d) Bishan Singh Bedi
Ans. b
Explanation: Indian cricketers in list- Bishan Singh Bedi, Kapil Dev, Kapil Dev

29. 2016 Olympic Games will be held at
(a) Chicago
(b) Madrid
(c) Rio de Janeiro
(d) Tokyo
Ans. c
Explanation: Rio de Janeiro has been chosen as the venue for the 2016 Olympic Games. This is the first time the Olympics are being held in a South American country. Rio de Janeiro beat Tokyo and Chicago among other cities to successfully bid for the hosting.


30. The mascot of the 19th Common-Wealth Games, to be held in 2010 in New Delhi is
(a) Bhaloo (b) Cheetah
(c) Chital (d) Shera
Ans. d
Explanation: Shera, mascot of the XIX Commonwealth Games 2010 Delhi, is the most visible face of the XIX Commonwealth Games 2010 Delhi. His name comes from the Hindi word Sher – meaning tiger.

31. The 'Man of the Series' in the triangular series of the Idea Cup 2010 concluded on 13th January, 2010 is
(a) M.S. Dhoni
(b) Kumar Sangakkara
(c) Virat Kohli
(d) Virender Sehwag
Ans. b
Explanation:
The 'Man of the Series' in the triangular series of the Idea Cup 2010 concluded on 13th January, 2010 is Kumar Sangakkara.

32. The atmosphere exerts enormous pressure on us. But, we do not feel it because
(a) our blood exerts a pressure slightly more than that of the atmosphere.
(b) We are used to it
(c) our bones are very strong and can withstand the pressure.
(d) the surface area of our head is very small.
Ans. a
Explanation:
The atmosphere exerts enormous pressure on us. But, we do not feel it because our blood exerts a pressure slightly more than that of the atmosphere.

33. In which of the following industries is mica used as a raw material?
(a) Iron and Steel
(b) Toys
(c) Glass and Pottery
(d) Electrical
Ans. d
Explanation: Mica has a high dielectric strength and excellent chemical stability, making it a favoured material for manufacturing capacitors for radio frequency applications. It has also been used as an insulator in high voltage electrical equipment, and between the bars of commutators in Direct Current motors and generators.

34. The micro-organism which is associated with the production of Bt cotton is a
(a) Fungus
(b) Bacterium
(c) Blue green Alga
(d) Virus
Ans. b
Explanation: Mahyco (Maharashtra Hybrid Seed Company), in collaboration with Monsanto, has introduced Bt cotton technology into India. Bt cotton carries the Cry1Ac gene derived from the common soil bacterium Bacillus thuringiensis var. kurstaki, which results in the expression of the Cry1Ac protein that confers resistance to insect pests—notably Helicoverpa armigera, commonly referred to as American Bollworm.

35. Which of the following leguminous plant is also a petro-plant?
(a) Pigeon-pea (b) Pea
(c) Gram (d) Pongam
Ans. d
Explanation: Pongamia pinnata is a deciduous legume tree and used for anti-inflammatory, anti-plasmodial, anti-nonciceptive, anti-hyperglycaemic, anti-lipidperoxidative, anti-diarrhoeal, anti-ulcer, anti-hyperammonic and antioxidant. Its oil is a source of biodiesel. It has also alternative source of energy, which is renewable, safe and non-pollutant.

36. If an apple is releasted from an orbiting spaceship, it will
(a) fall towards the Earth
(b) move at a lower speed
(c) move along with the spaceship at the same speed
(d) move at a higher speed
Ans. c
Explanation:
If an apple is releasted from an orbiting spaceship, it will move along with the spaceship at the same speed.

37. Consider the following statements:
Assertion (A): Space Based Solar Poser (SBSP), it is suggested be made a national goal.
Reason (R) : Supply of SBSP is 99% uninterrupted throughout the year, besides the enormity of energy availability.
Select the correct answer using the codes given below:
Codes:
(a) Both (A) and (R) are true and (R) is the correct explanation of (A).
(b) Both (A) and (R) are true, but (R) is not the correct explanation of (A)
(c) (A) is true, but (R) is false.
(d) (A) is false, but (R) is true.
Ans. a(?)
Explanation:
Space-based solar power (SBSP) is a system for the collection of solar power in space, for use on Earth. SBSP differs from the usual method of solar power collection in that the solar panels used to collect the energy would reside on a satellite in orbit, often referred to as a solar power satellite (SPS), rather than on Earth's surface. In space, collection of the Sun's energy is unaffected by the various obstructions which reduce efficiency or capacities of Earth surface solar power collection.

38. Computer virus is a
(a) fungus
(b) bacterium
(c) IC 7344
(d) Software program
Ans. d
Explanation: A computer virus is a computer program that can copy itself and infect a computer. The term "virus" is also commonly but erroneously used to refer to other types of malware, adware, and spyware programs that do not have the reproductive ability.

39. Match List-I with List-II and select the correct answer using the codes given below the lists.
List-I List-II
A. Arithant 1. American Cryogenic Rocket
B. Awacs 2. Anti tank missile
C. Atlas 3. Israel developed
Centaur Falcon for Air
Force
D. Nag 4. Atomic submarine
Codes:
A B C D
(a) 4 3 2 1
(b) 4 3 1 2
(c) 3 4 1 2
(d) 3 4 2 1
Ans. b
Explanation:
The Atlas-Centaur was an American expendable launch system derived from the SM-65 Atlas missile. It was the first rocket to use cryogenic fuel; the Centaur stage burned liquid hydrogen and liquid oxygen.
India’s first nuclear-powered submarine INS Arihant , becoming only the sixth country in the world to do so.

40. Salk's vacine is connected with which one of the following diseases?
(a) Small pox (b) Tetanus
(c) T.B. (d) Polio
Ans. d
Explanation: The Salk vaccine, or inactivated poliovirus vaccine (IPV), is based on three wild, virulent reference strains, type 1 poliovirus, type 2 poliovirus, and type 3 poliovirus The injected Salk vaccine confers IgG-mediated immunity in the bloodstream, which prevents polio infection.

41. Match List-I with List-II and select the correct answer using the codes given below the lists.
List-I List-II
(Disease) (Cause)
A. Marasmus 1. Prolonged
Starvation
B. Kwashiorkor 2. Protein
Deficiency
C. T.B. 3. Bacterial
Infection
D. Hepatitis B 4. Viral Infection

Codes:
A B C D
(a) 1 2 3 4
(b) 2 1 3 4
(c) 4 2 3 1
(d) 2 4 1 3
Ans. a
Explanation:
Marasmus is a form of severe protein-energy malnutrition characterized by energy deficiency. Kwashiorkor (pronounced /kwɑːʃiˈɔrkər/) is an acute form of childhood protein-energy malnutrition characterized by edema, irritability, anorexia, ulcerating dermatoses, and an enlarged liver with fatty infiltrates. Kwashiorkor was thought to be caused by insufficient protein consumption but with sufficient calorie intake, distinguishing it from marasmus.

42. Which one of the following is not property matched?
(a) Gene splicing & recombinant DNA technology —Genetic Engineering

(b) A diagnostic test to detect the presence or absence or genetic disorders in unborn child - Amniocentesis

(c) A process by which living organisms break down complex matter into simpler constitutents - Biodegradation

(d) An inbuilt time- keeping system in all organisms
- Bio-mass
Ans. d

43. Consider the following statements:
Assertion (A): If Ice collects on the freeze the cooling in the refrigerator is affected adversely.
Reason (R) : Ice is a poor conductor.
Select the correct answer using the codes given below:
Codes:
(a) Both (A) and (R) are true and (R) is the correct explanation of (A).
(b) Both (A) and (R) are true, but (R) is not the correct explanation of (A).
(c) (A) is true, but (R) is false.
(d) (A) is false, but (R) is true.

Ans. b
Explanation:
Early freezer units accumulated ice crystals around the freezing units. This was a result of humidity introduced into the units when the doors to the freezer were opened. This frost buildup required periodic thawing ("defrosting") of the units to maintain their efficiency.

44. What type of electromagnetic radiation is used in the remote control of a television
(a) infrared
(b) Ultraviolet
(c) Visible
(d) None of these
Ans. a
Explanation: Most of electronic remotes communicate to their respective devices via infrared (IR) signals and a few via radio signals. Television IR signals can be mimicked by a universal remote, which is able to emulate the functionality of most major brand television remote controls.

45. Which one of the following statements is not true for a person suffering from hypermetropia?
(a) The person can see far objects distinctly.
(b) The focal length of the lens is large.
(c) The image of the close objects is focussed behind the retina.
(d) A concave lens is used to correct this defect.
Ans. d
Explanation:
Also called far-sightedness, or far-sighted. Rays of light reflected off objects in front of the eye are focused behind the retina. hypermetropia caused by an imperfection in the eye (often when the eyeball is too short or The focal length of the lens is large). It may be corrected with convex lenses in eyeglasses.

46. "Endoscope" used by Doctors for examining the inside of the patient's stomach, works on the principle of
(a) reflection of light
(b) dispersion of light
(c) total internal reflection of light
(d) refraction of light
Ans. c
Explanation: "Endoscope" used by Doctors for examining the inside of the patient's stomach, works on the principle of total internal reflection of light

47. An enzyme which helps in the digestion of protein is
(a) urease (b) sulfatase
(c) trypsin (d) protease
Ans. d (most correct answer)
Explanation: Trypsin is a serine protease found in the digestive system of many vertebrates, where it hydrolyses proteins.
A protease is any enzyme that conducts proteolysis, that is, begins protein catabolism by hydrolysis of the peptide bonds that link amino acids together in the polypeptide chain forming the protein.

48. Methane is present in the atmosphere of
(a) Moon (b) Sun
(c) Jupiter (d) Mars
Ans. c
Explanation:
Surface of Jupiter is made up of many gases like hydrogen, Helium and methan.

49. Hydrogen bomb is based on the principle of
(a) controlled fusion reaction
(b) uncontrolled fusion reaction
(c) controlled fission reaction
(d) uncontrolled fission reaction
Ans. a
Explanation: Hydrogen bomb, a thermonuclear weapon is based on the principle of uncontrolled fusion reaction.

50. Match List-I with List-II and select the correct answer using the codes given below the lists:
List-I List-II
(Naturally (Elements present)
occurring
substance)
A. Diamond 1. Calcium
B. Marble 2. Silicon
C. Sand 3. Aluminum
D. Ruby 4. Carbon

Codes:
A B C D
(a) 3 1 2 4
(b) 4 2 1 3
(c) 2 1 3 4
(d) 4 1 2 3
Ans. d

51. Which one of the following compounds is used as a sedative?
(a) Potassium bromide
(b) Calcium chloride
(c) Ethyl alcohol
(d) Phosphorus trichloride
Ans. a
Explanation: Potassium bromide (KBr) is a salt, widely used as an anticonvulsant and a sedative in the late 19th and early 20th centuries. Its action is due to the bromide ion (sodium bromide is equally effective). Potassium bromide is presently used as a veterinary drug, as an antiepileptic medication for dogs and cats.

52. The hypo solution used in photography is the aqueous solution of
(a) Sodium thiosulphate
(b) Sodium tetrahionate
(c) Sodium sulphate
(d) ammonium per sulphate
Ans. a
Explanation: The hypo solution used in photography is the aqueous solution of Sodium thiosulphate. It is used as an antidote to cyanide poisoning. It is also widely used in rural area as a topical antifungal agent for tinea versicolor.

53. Accumulation of which one of the following in the muscles leads to fatigue?
(a) Lactic acid
(b) Benzoic acid
(c) Pyruvic acid
(d) Uric acid
Ans. a
Explanation:
. Accumulation of Lactic acid in the muscles leads to fatigue.

54. Which one of the following had supported the non-cooperation movement, but could not see its upshots?
(a) Bal Gangadhar Tilak
(b) Lala Lajpat Rai
(c) Motilal Nehru
(d) Chitranjan Das
Ans. a
Explanation:
Bal Gangadhar Tilak had supported the non-cooperation movement, but could not see its upshots. Bal Gangadhar Tilay Gandhijee.k passed away on 1 Aug 1920. His place was soon taken.

55. Which of the following is a substance abundantaly available in the sea and administered in a certain deficiency disease?
(a) Iron (b) Vitamin A
(c) Fluorine (d) Iodine
Ans. d
Explanation:
Iodine is a substance abundantaly available in the sea and administered in a certain deficiency disease.

56. In countries where polished rice is the mean cereal in their diet, people suffer from
(a) Pellagra (b) beri-beri
(c) scurvy (d) osteomalacia
Ans. b
Explanation:
In countries where polished rice is the mean cereal in their diet, people suffer from beri beri because polices rice is deficient in water soluble vitamins specially vitamin B1.

57. Which one of the following silver salts is used for producing artificial rains?
(a) Silver chloride
(b) Silver bromide
(c) Silver nitrate
(d) Silver iodide
Ans. d
Explanation:
Silver iodide and dry ice are used for producing artificial rains

58. With which one of the following movements was Aruna Asaf Ali associated?
(a) Khilafat Movement
(b) Non Cooperative Movement
(c) Civil Disobedience Movement
(d) Quit India Movement
Ans. d
Explanation:
Aruna went to prision during the 1930 and 1932 Civil Disobedience Movement and in 1040 in connection with Gandhijee’s call for Individual Satyagraha. She was elected the first mayer of Delhi in 1958.

59 The official historian of India's struggle for independence was
(a) R.C. Majumdar
(b) Tarachand
(c) V.D. Savarkar
(d) S.N. Sen
Ans. d (?)
Explanation:
The official historian of India's struggle for independence was Tarachand.

60. Who among the following was not associated with the Home Rule Movement?
(a) C.R. Das
(b) S. Subramaniya Iyer
(c) Annie Besant
(d) B. G. Tilak
Ans. a
Explanation:
Two Home Rule Leagues were started in 1915-16, one under leadership of Lokamanya Tilak and the other under the leadership of Annie Besant and S. Subramaniya.

61. Read the following events connected with Indian National Movement and find the correct chronological order of the events from the codes given below:
(a) Cripps Mission
(b) Cabinet Mission Plan
(c) Quit India Movement
(d) Wavell Offer
Code:
(a) 1,3,4,2 (b) 1,2,3,4
(c) 1,3,2,4 (d) 4,3,2,1
Ans. a
Explanation:
Cripps Mission- In March 1942, a mission headed by Stafford Cripes was sent to India with Constitutional proposal to seek Indian supportfor the war.
Quit India Movement- After Cripps departure, Gandhi framed a resolution calling for British withdrawal and a non violent non cooperative movement against any Japanese invasion. The CWC meeting at Wardha July 14, 1942 accepted the idea for struggle.
Wavell Offer- The idea was to reconstruct the governor general’s executive council pending the the preparation of a new constitution.For this porosr, a conference was convened by The viceriy, Lord Wavell, at Simla in June 1945.
Cabinet Mission Plan- The Atttlee Government in Feb 1946 the decision to send a high powered mission of three British Cabinet members to India to find out the ways and means for a regociated, peacefull tranfer of power to India.

62. Given below are two statements labelled as:
Assertion(A): The Congress boycotted the Simon Commission.
Reason(R): The Simon Commission did not have a single Indian member.
In the context of the above, which one of the following is correct.
(a) Both (A) and (R) are true and (R) is the correct explanation of (A).
(b) Both (A) and (R) are true, but (R) is not the correct explanation of (A).
(c) (A) is true, but (R) is false.
(d) (A) is false, but (R) is true.
Ans. a
Explanation:
Simon Commission came in 1928 to explore possibility of further constitutional advance and bycotted by Indians because no indians represented in Commission.

63. Who out of the following had told "Destruction is the best method of dealing with the foreign clothes?
(a) Ravindra Nath Tagore
(b) Mahatma Gandhi
(c) Chitranjan Das
(d) Subhas Chandra Bose
Ans. b
Explanation:
Mahatma Gandhi had told "Destruction is the best method of dealing with the foreign clothes.

64. Which one of the following is not correctly matched?
(a) larry Collins - Freedom at
and Dominique Midnight
Lapierre
(b) Durga Das - India from
Curzon to
Nehru and
After
(c) K.K. Aziz - The Man
Who Divided
India
(d) Maulana Abul - India Wins
Kalam Azad Freedom
Ans.
Explanation: c
THE MAN WHO DIVIDED INDIA — An Insight into Jinnah's Leadership and its Aftermath: Rafiq Zakaria

65. Where was Mahatma Gandhi when a raid was made by Congress Volunteers on Dharsana Salt Depot?
(a) In Yervada Jail
(b) In Sabarmati Jail
(c) In Agha Khan Palace Poona
(d) In Ahmadnagar Fort Jail
Ans. a
Explanation:
Mahatma Gandhi was in Yervada Jail when a raid was made by Congress Volunteers on Dharsana Salt Depot. On May 1930, Sarojini Naidu, Imam Sahib and Manilal took up the unfnished task of leading a raid on Dharsana Salt Depot.

66. Match the peasants revolts of the 19th century with their respective areas
(a) Kuki revolt 1. Punjab
(b) Kuka revolt 2. Bengal
(c) Pabna Peasant revolt 3. Bihar
(d) Birsa Munda revolt 4. Tripura
Choose your answer with the help of given code:
Codes:
A B C D
(a) 4 2 1 3
(b) 2 3 1 4
(c) 4 1 3 2
(d) 4 1 2 3
Ans. d
Explanation:
Kuki revolt-------------------Tripura
Kuka revolt------------------ Punjab
Pabna Peasant revolt------- Bengal
Birsa Munda revolt----------- Bihar

67. Match List-I with List-II and select the correct answer using the codes given below the lists:
List-I List-II
A. Jhansi 1. Moulavi Ahmad
Shah
B. Lucknow 2. Azimullah Khan
C. Kanpur 3. Begum Hazrat
Mahal
D. Faizabad 4. Rani Laxmi Bai
Codes:
A B C C
(a) 4 3 2 1
(b) 4 2 3 1
(c) 3 4 2 1
(d) 1 2 3 4
Ans. a
Explanation:
Jhansi- Rani Laxmi Bai
Lucknow- Begum Hazrat Mahal
Kanpur- Azimullah Khan
Faizabad-Moulavi Ahmad Shah

68. The entry of woman as a bhikshuni into the Buddhist Sangha was allowed by Gautam Buddha at
(a) Sravasti (b) Vaisali
(c) Rajagriha (d) Kushinagar
Ans. b
Explanation:
The entry of woman as a bhikshuni into the Buddhist Sangha was allowed by Gautam Buddha at Vaisali. Buddha gave women permission to join the order at Vaisali after acceptance of the eight gurudhamma-eight important rules by his foster-mother Mahapajapati Gotami, who became the first ordained bhikkhuni.

69. Choose the correct pair
(a) Ellora Caves - Saka
(b) Mahabalipuram - Rashtrakutas
(c) Meenakshi Temple- pallavas
(d) Khajuraho - Chandellas
Ans. d
Explanation:
Ellora is an archaeological site, 30 km (19 mi) from the city of Aurangabad in the Indian state of Maharashtra built by the Rashtrakuta rulers.
Mahabalipuram is a town in Kancheepuram district in the Indian state of Tamil Nadu. Mahabalipuram was a 7th century port city of the South Indian dynasty of the Pallavas.
Meenakshi Temple is an historic Hindu temple located in the holy city of Madurai, Tamil Nadu, India. It is dedicated to Lord Shiva and his consort, Goddess Parvati. Meenakshi temple were built during the Nayak rule.

70. Budha is depicted on the coins of
(a) Wima Kadphises
(b) Kanishka
(c) Nahapana
(d) Budh Gupta
Ans. b
Explanation:
Budha is depicted on the coins of Kanishka. Several of The Buddhist coins of Kanishka display Kanishka himself on the obverse, and the Buddha standing on the reverse.

71. Put the following events in chronological order and choose your answer with the help of given code:
A. Formation of an interim Government
B. The arrival of the Cabinet Mission
C. Muslim League launches Direct Action
D. Jinnah's wrecking of the Shimla Conference
Codes:
(a) B D C A (b) D B C A
(c) A B D C (d) D B A C
Ans. a
Explanation:
The arrival of the Cabinet Mission
Jinnah's wrecking of the Shimla Conference
Muslim League launches Direct Action
Formation of an interim Government

72. In which of the follwing movements did Mahatma Gandhi make the first use of hunger strike as a weapon?
(a) Non-Cooperation Movement
(b) Rowlatt Satyagraha
(c) Ahmedabad Strike
(d) Bardoli Satyagraha
Ans. c
Explanation:
Ahmedabad Strike- first use of hunger strike as a weapon. Gandhi intervened in a dispute between mill owners of Ahmedabad and workers over the issue of discontinuation of the plague bonus. Gandhi asked the workers to go a strike and demand a 35% increase in wage.

73. Mention of which of the following rivers in the Rigveda suggests the Aryan's connection with Afghanistan?
(a) Asikni
(b) Parushni
(c) Kubha, Kramu
(d) Vipas, Sutudri
Ans. c
Explanation:
Rigvedic aryans were wll aquainted with region of Afghanistan. Rigveda mentions Afghanistan’s rivers Kubha, Kramu, Gomati and Suvasti.

74. The city plan of ancient Shravasti is in the shape
(a) Circular
(b) Crescentic
(c) Triangular
(d) Rectangular
Ans. d(?)
Explanation:
The city plan of ancient Shravasti is in the shape rectangular.

75. Where is the wild ass sasnctuary?
(a) U.P. (b) Assam
(c) Gujarat (d) Rajasthan
Ans. c
Explanation:
Indian Wild Ass Sanctuary also known as the Wild Ass Wildlife Sanctuary is located in the Little Rann of Kutch in the Gujarat state of India.

76. Kissan Bahi Yojana was started in Uttar Pradesh in
(a) 1970 (b) 1975
(c) 1990 (d) 1992
Ans. d
Explanation:
Kissan Bahi Yojana was started in Uttar Pradesh in 1992.

77. Yapaniya was a school of
(a) Buddhism (b) Jainism
(c) Saivism (d) Vaisnavism
Ans. b
Explanation:
Yapaniya was a Jain order in western Karnataka which is now extinct.

78. The first Gupta ruler to assume the title of "Param Bhagawata" was
(a) Chandragupta I
(b) Samudragupta
(c) Chandragupta II
(d) Srigupta
Ans. c
Explanation:
Chandragupta II was described as Param Bhagwata.

79. Panini and Patanjali are the renowned names in the literary history of ancient India. Under which dynasty did they flourish?
(a) Pushyabhukti (b) Kushanas
(c) Sungas (d) Guptas
Ans. c
Explanation:
Patañjali is the compiler of the Yoga Sutras, an important collection of aphorisms on Yoga practice, and also the author of the Mahābhāṣya, a major commentary on Pāṇini's Ashtadhyayi. Panini and Patanjali flourished Under Sungas dynasty.

80. The great Jain Scholar Hemachandra adorned the court of
(a) Amoghavarsha
(b) Kumarapala
(c) Jaysimha Siddharaja
(d) Vidyadhara
Ans. b
Explanation:
He was born in Dhandhuka, Gujarat. At the time, Gujarat was ruled by the Solanki dynasty. Hemachandra rose to prominence under the reign of Siddharaj Jaysinh I, and was an advisor to his successor Kumarapala (1143–1173). During Kumarapala's reign, Gujarat became a reputed center of culture.

81. The three age system - Stone,Bronze and Iron from the collection of Copenhagen museum was coined by
(a) Thomson (b) Lubbock
(c) Taylor (d) Childe
Ans. a
Explanation:
The three-age system is the periodization of human prehistory into three consecutive time periods, named for their respective predominant tool-making technologies:The Stone Age,The Bronze Age,The Iron Age. Its formal introduction is attributed to the Danish archaeologist Christian Jürgensen Thomsen in the 1820s in order to classify artifacts in the collection which later became the National Museum of Copenhagen, Denmark.

82. Which musical instrument was played by the Mughal Emperor Aurganzeb?
(a) Sitar
(b) Pakhawaj
(c) Veena
(d) None of the above
Ans. c
Explanation:
Mughal Emperor Aurganzeb was an accomplished veena player.The largest number of books on classical Indian music in Persian were written during Aurganzeb’s reign.

83. Which one of the following pairs is not correctly matched?
(a) Babar - Battle of Khanwa
(b) Humayun - Battle of Chausa
(c) Akbar - Battle of Haldighati
(d) Jahangir - Battle of Balkh
Ans. d
Explanation:
The Battle of Khanwa was the second in a series of three major battles, victories in which gave Zahir ud-Din Babur overlordship over North India.
When Humayun turned his attention from the east Sherkhan attacked him at a place called Chausa. In this battle of Chausa in 1539, Humayun was defeated and he escaped to Agra.
The Battle of Haldighati was fought between the Mughal Empire and the forces of Mewar on June 21, 1576 at Haldighati in Rajasthan, India.It was a decisive victory for the Mughal Emperor Jalal ud-Din Muhammad Akbar's general Raja Maan Singh against the Maharana Pratap Singh of Mewar

84. Match List-I with List-II and select the correct answer from the codes given below the lists
List-I List-II
(Monuments) (Builders)
A. Alai Darwaja, 1. Alauddin
Delhi Khalji
B. Buland Darwaja 2. Akbar
Fatehpur Sikri
C. Moti Masjid, Agra 3. Shahjahan
D. Moti Masjid, Delhi 4. Aurangzeb
Codes:
A B C C
(a) 1 2 3 4
(b) 3 2 1 4
(c) 4 1 2 3
(d) 1 4 3 2
Ans. a
Explanation:
The Alai Darwaza is a magnificent gateway built by Ala-ud-din Khilji of the Delhi Sultanate.
The Buland Darwaza is a grand gateway located in Fatehpur Sikri near Agra. This triumphal arch was built by the Mughal Emperor Akbar, when he defeated the king of Khandesh or Gujarat in 1573.
The Moti Masjid in Agra was built by Shah Jahan. During the rule of Shah Jahan. Moti Masjid's architectural features are quite similar to that of the Saint Basil's Cathedral in Moscow.
The Moti Masjid is a large white marble mosque built by the Mughal emperor Aurangzeb at the Red Fort complex in Delhi, India,

85. In U.P. the first 'Biotechnology Park' was established at
(a) Lucknow (b) Noida
(c) Agra (d) Kanpur
Ans. a
Explanation:
In U.P. the first 'Biotechnology Park' was established at Lucknow.

86. Who had composed the 'Gita Govinda'?
(a) Dhoyi
(b) Govardhanacharya
(c) Jayadeva
(d) Lakshmana Sen
Ans. c
Explanation:
The Gita Govinda (Sanskrit गीत गोविन्द) (Song of Govinda) is a work composed by the 12th-century poet, Jayadeva, who was born in Kenduli Sasan near Puri in Orissa. This work has been of great importance in the development of the bhakti traditions of Hinduism.

87. Which medieval King of India introduced the "Iqkta system"?
(a) Iltutmish
(b) Balban
(c) Alauddin Khalji
(d) None of the above
Ans. a
Explanation:
In the Islāmic empire of the Caliphate, land granted to army officials for limited periods in lieu of a regular wage. The iqṭāʿ system was established in the 9th century ad to relieve the state treasury when insufficient tax revenue and little booty from campaigns made it difficult for the government to pay army salaries. Iltutmish King of India introduced the "Iqkta system" in India.

88. Jain temple of Abu is made of
(a) Sandstone (b) Lime Stone
(c) Granite (d) marble
Ans. d
Explanation:
Jain Temple of Abu is one of the finest Jain temple known world over for its extraordinary architecture and marvelous marble stone carvings.

89. Which among the following organisations supportecd the Suddhi movement?
(a) Arya Samaj
(b) Brahma Samaj
(c) Deva Samaj
(d) Prarthana Samaj
Ans. a
Explanation:
The Arya Samajis were threatened with the invention of a mode of conversion and wanted to present themselves as the protectors of the entire Hindu community. In this regard the Suddhi Movement started.

90. The institution of local self government got a fillip during the Viceroyalty os
(a) Lord Mayo
(b) Lord Ripon
(c) Lord Dufferin
(d) Lord Curzon
Ans. b
Explanation:
In the late 19th century, the British Government initiated the concept of Local Self Government. When Lord Ripon became the Viceroy of India and provided notable contribution to the development of Local Government in the country. Lord Ripon is considered to be the founding father of urban local government.

91. With which Uprising is Mangal Pandey associated?
(a) Barrackpur
(b) Meerut
(c) Delhi
(d) None of the above
Ans. a
Explanation:
Even before the outbreak at Meerut, Mangal Pande had become a martyr at Barrackpore.

92. With one of the following is correctly matched?
A. Khuldabad- Tomb of Shaikh
Salim Chisti
B. Fatehpur Tomb of Itimad-
Sikri ud-Daula
c. Agra Tomb of
Aurangzeb
d. Delhi Tomb of Abdur- Rahim Khan-i- khanan
Ans. d
Explanation:
The tomb of Shaikh Salim was originally built with red sandstone but later converted into a beautiful marble mausoleum. The mausoleum was built by Akbar as a mark of his respect and reverence for the saint. Tomb of Sheikh Salim Chishti is located within the Jama Masjid complex of Fatehpur Sikri, U.P..
Aurangzeb's tomb at Khultabad in Aurangabad at Mahrashtra state.
Itmad-ud-Daula's is a Mughal mausoleum in the city of Agra in the Indian state of Uttar Pradesh.

93. The Kirtistambha at Chittor was built by
(a) Rana Sanga
(b) Rana Kumbha
(c) Rana Pratap
(d) Rajna Udai Singh
Ans. b ( most correct answer)
Explanation:
The Kirtistambha at Chittor was built by Rana Kumbha(?).Kirti Stambh or "Tower of Fame" is situated at Chittorgarh fort in Rajasthan, India. The 22 metre high tower was built by a Bagherwal Jain merchant Jijaji Kathod in 12th century.Vijay Stambha or "Tower of Victory" is an imposing structure located in Chittorgarh fort in Rajasthan, India. It was constructed by Mewar king Rana Kumbha between 1442 AD and 1449 AD to commemorate his victory over the combined armies of Malwa and Gujarat led by Mahmud Khilji .

94. Who among the following rulers is famous as "Prithvi Raj Chauhan"?
(a) Prithvi Raj I
(b) Prithvi Raj II
(c) Prithvi Raj III
(d) None of the above
Ans. c
Explanation:
Prithvi Raj III, commonly known as Prithvi Raj Chauhan, who ruled the kingdom of Ajmer and Delhi in northern India during the latter half of the 12th century.

95. Arrange the following million cities of Uttar Pradesh in ascending order of their population size.
1. Agra 2. Allahabad
3. Meerut 4. Lucknow
use the code given below to select the correct answer.
(a) 1,3,2,4 (b) 4,2,1,3
(c) 2,1,4,3 (d) 2,3,1,4
Ans. d
Explanation: million cities of Uttar Pradesh in descending order of their population size.
1. Kanpur
2. Lucknow
3. Agra
4. Ghaziabad
5. Meerut
6. Varanasi
7. Allahabad

96. The author of the book "India Divided" was
(a) Maulana Abul Kalam Azad
(b) Dr. Rajendra Prasad
(c) Narendra Deva
(d) Asaf Ali
Ans. b
Explanation:
The author of the book "India Divided" was Dr. Rajendra Prasad.

97. Match List-I with List-II and select the correct answer from the codes given below the lists
List-I List-II
(Authors) (Works)
A. Bankim Chandra 1. Anand Math
Chatterji
B. Michael 2. Captive Lady
madhusudan Das
C. Ravindra Nath 3. Gora
Tagore
D. Sarojini Naidu 4. The Broken Wing
Codes:
A B C C
(a) 1 2 3 4
(b) 2 3 4 1
(c) 1 4 2 3
(d) 4 1 3 2
Ans. a
Explanation:
Anand Math- is a Bengali novel, written by Bankim Chandra Chatterji.
The Broken Wing- Sarojini Naidu
Gora- Ravindra NathTagore

98. In which of the Indian Provinces the first Communist Government was established?
(a) Tamilnadu
(b) Andra Pradesh
(c) Kerala
(d) West Bengal
Ans. c
Explanation:
The first communist government assumed power in Kerala in 1957, after winning a majority in the state elections. Thus the EMS Namboodiripad government of 1957 set a a new example not only for the communist parties of other Indian states but also of the rest of the world to emulate.

99. Which one of the following biosphere reserves is not ioncluded ion the list of world network of biosphere reserves of UNESCO?
(a) Simlipal
(b) Sunderban
(c) Gulf of Mannar
(d) Nilgiri
Ans. a (most correct answer)
Explanation:
• Nilgiri Biosphere Reserve
• Nanda Devi Biosphere Reserve
• Sundarbans national park
• Gulf of Mannar Biosphere Reserve
• Pachmarhi Biosphere Reserve
• Nokrek Biosphere Reserve
• Simlipal Biosphere Reserve
The last three are among the 22 new sites recently added by UNESCO to its World Network of Biosphere Reserves. Their inclusion raises the number of Indian sites on the World list to seven, from a total of 15 Biosphere Reserves across the subcontinent. Recently India has added one more area as bsr namely cold desert of Himachal Pradesh taking the total count to 16.

100. As per Census 2001, the class I cities of India claim a share of the total urban population of
(a) 40.40% (b) 56.50%
(c) 65.20% (d) 62.32%
Ans. c
Explanation:
As per Census 2001, the class I cities of India claim a share of the total urban population of 65.20%.
Note- As per Census 2001, Urban population as percentage to total population=27.82
As per Census 2001, the class I cities of India claim 178 millions population.

101. Which megacity of India generates the largest solid waste per capital annually?
(a) Bangalore (b) Chennai
(c) Delhi (d) Mumbai
Ans. d
Explanation:
Mumbai generates the largest solid waste per capital annually.

102. Today the largest contributor to carbon dioxide (CO2) emissions is
(a) China (b) Japan
(c) Russia (d) U.S.A.
Ans. a
Explanation:
China Overtakes U.S. as No. 1 Emitter of Carbon Dioxide.

103. Which one of the following is the most important factor responsible for decline of bio-diversity?
(a) Genetic assimilation
(b) Controlling predators
(c) Destruction of habitat
(d) Controlling pests
Ans. c
Explanation:
Destruction of habitat following is the most important factor responsible for decline of bio-diversity.

104. Which one of the following States of India has the highest rural literacy rate?
(a) Maharashtra (b) Manipur
(c) Goa (d) Gujarat
Ans. c

105. The minimum percentage of population below the poverty limit is recorded in
(a) Jammu and Kashmir
(b) Punjab
(c) Haryana
(d) Goa
Ans. a

106. Jhooming is practised by
(a) The Bhotias
(b) The Khasis
(c) The Santhals
(d) The Todas
Ans. b
Explanation:
Jhum cultivation or shifting agriculture used to be practiced by the tribal groups in the north-eastern states of India like Arunachal Pradesh, Meghalaya, Mizoram and Nagaland. This system often involves clearing of a piece of land followed by several years of wood harvesting or farming until the soil loses fertility. The Khasis inhabit the eastern part of Meghalaya and practice Jhum cultivation.

107. Consider the following statements:
Assertion (A) : Madhya Pradesh is called the Ethiopia of India
Reason (R) : Its hallmarks are excessive infantile mortality and malnutrition.
Select the correct answer using the codes given below:

Codes:
(a) Both (A) and (R) are true and (R) is the correct explanation of (A).
(b) Both (A) and (R) are true, but (R) is not the correct explanation of (A)
(c) (A) is true, but (R) is false.
(d) (A) is false, but (R) is true.
Ans. a
Explanation:
Madhya Pradesh seems to have emerged as the poverty bowl of the Asian region. It has not only surpassed its neighbouring states like Chhattisgarh, but has outdone even Ethiopia in terms of hunger, according to the Planning Commission.

108. The UNESCO has given international recognition to
(a) Nilgiri Biosphere
(b) Nanda Devi Biosphere
(c) Manas Biosphere
(d) Simlipal Biosphere
Select the correct answer from the following code:
Codes:
(a) 1 and 2 (b) 1 and 3
(c) 2 and 3 (d) 2 and 4
Ans. a
Explanation:
The UNESCO has given international recognition to Nilgiri Biosphere and Nanda Devi Biosphere.

109. Which one of the following lakes has been recently included under National Lake Conservation Project?
(a) Bhimtal (b) Pulicat
(c) Ooty (d) Sambhar
Ans. a
Explanation:
The Ministry of Environment & Forests, Government of India has initiated a Centrally Sponsored Scheme called the National Lake Conservation Plan (NLCP), since June, 2001, on 70:30 cost sharing basis between the Central Government and the respective State Government, with the objective of restoring and conserving the polluted and degraded lakes in urban and semi urban areas in the country. Bhimtal has been recently included under National Lake Conservation Project.

110. The population of U.P. exceeds that of
1. Bangladesh 2. Brazil
3. Pakistan 4. Indonesia
Select the correct answer from the following code:
(a) 1 and 2 (b) 1 and 3
(c) 2 and 3 (d) 2 and 4
Ans. b (most correct answer)
Explanation:
1. Bangladesh-156 millions
2. Brazil-198 millions
3. Pakistan-176 millions
4. Indonesia-240 millions
UP-166 millions

111. If interest payment is added to primary deficit, it is equivalent to
(a) Budget deficit
(b) Fiscal deficit
(c) Deficit financing
(d) Revenue deficit
Ans. b
Explanation:
If interest payment is added to primary deficit, it is equivalent to Fiscal deficit. Concept of Fiscal deficit is being used since fiscal 1997-98 in India.

112. Consider the following statements and state which is/are correct?
1. The sub-prime crisis which hit the U.S. economy was caused by sudden increase in oil prices.
2. The crisis ld to default in home-loan repayment.
3. It led to failure of some U.S. banks.
4. Sub prime crisis caused crash in Indian Stock Market.
Choose your answer from given code.
Codes:
(a) 1 only
(b) 2 and 4 only
(c) 2, 3 and 4 only
(d) 1, 2, 3 and 4
Ans. c
Explanation:
The subprime mortgage crisis is an ongoing real estate crisis and financial crisis triggered by a dramatic rise in mortgage delinquencies and foreclosures in the United States, with major adverse consequences for banks and financial markets around the globe.

113. In Uttar Pradesh, recognized Stock Exchange is at
(a) Lucknow (b) Kanpur
(c) Varanasi (d) Ghaziabad
Ans. b
Explanation:
In Uttar Pradesh, recognized Stock Exchange is at Kanpur.

114. National Renewal Fund was constituted for the purpose of
(a) providing pension for retiring employees
(b) social security
(c) rural reconstruction
(d) restructuring & modernization of industries.
Ans. d
Explanation:
In order to protect the interest of workers, Government had set up a National Renewal Fund (NRF) in 1992. National Renewal Fund was constituted for the purpose of restructuring & modernization of industries.

115. Open-market operations of Reserve Bank of India refer to
(a) trading in securities
(b) auctioning of Foreign Exchange
(c) transaction in gold
(d) none of the above
Ans. a
Explanation:
Open market operations are the means of implementing monetary policy by which a central bank controls the short term interest rate and the supply of base money in an economy, and thus indirectly the total money supply. This involves meeting the demand of base money at the target rate by buying and selling government securities, or other financial instruments.

116. An increase in CRR by the Reserve Bank of India results in
(a) decrease in debt of the government.
(b) reduction in liquidity in the economy.
(c) attracting more FDI in the country.
(d) more flow of credit to desired sectors.
Ans. b
Explanation:
An increase in CRR by the Reserve Bank of India results in reduction in liquidity in the economy. The Reserve Bank of India raised CRR of banks in April 2010 by 25 basis points which will suck out Rs. 12,500 crore in economy.

117. Special Economic Zone (SEZ) Act became effective in
(a) 2004 (b) 2005
(c) 2006 (d) 2007
Ans. c
Explanation:
Special Economic Zone (SEZ) Act, 2005, supported by SEZ Rules, have come into effect on 10th Feb 2006.Asia’s first EPZ was set up in Kandla, India in 1965.

118. In the 11th Five Year Plan, which of the following sectors accounts for maximum combined Central, State and U.T. expenditure?
(a) Transport
(b) Energy
(c) Agriculture and Rural Development
(d) Social sector
Ans. d
Explanation:
Social sector accounts for maximum combined Central, State and U.T. expenditure In the 11th Five Year Plan. Expenditure on social services by Centre and State as a proportion of total expenditure is 23.8 in 2009-10.

119. Which of the following provides the largest credit to agriculture and allied sectors?
(a) Cooperative Banks
(b) Regional Rural Banks
(c) Commercial Banks
(d) Cooperative and Regional Rural Banks jointly.
Ans. c
Explanation:
Share of Agriculture Credit
Commercial Bank-68 %
Regional Rural Banks-12%
Cooperative Banks-20%

120. Who had suggested an imposition of 'expenditure tax' in India for the first time?
(a) Kalecki
(b) Kaldor
(c) R.J. Chelliah
(d) Gautam Mathur
Ans. b
Explanation:
In order to widen the tax base, Kaldor in his tax reform proposal for India suggested the imposition of expenditure tax, wealth tax, gift tax and property tax.

121. Which one of the following writes is issued during the pendency of proceedings in a court?
(a) Mandamus
(b) Certiorari
(c) Prohibition
(d) Quo warranto
Ans. b
Explanation:
Certiorari is writes is issued during the pendency of proceedings in a court. It is issued by a higher court to a lower court or tribunal either to transfer a case pending with latter in a case. It is issued on the grounds of excess of Jurisdiction or lack of jurisdiction or error of law.

122. The word 'Secularism' was inserted in the Preamble of the Indian Constitution by
(a) 25th Amendment
(b) 42nd Amendment
(c) 44th Amendment
(d) 52nd Amendment
Ans. b
Explanation:
The word 'Secularism' was inserted in the Preamble of the Indian Constitution by 42nd Amendment of 1976. The Indian Constitution embodies the positive concept of Secularism.

123. Mid day Meal Scheme is financed and managed by
(a) Food and Civil Supply Department of State Governments
(b) Department of Consumer Affairs and Welfare
(c) Ministry of Programme Implementation.
(d) Ministry of Human Resource Development
Ans. d
Explanation:
Mid day Meal Scheme is financed and managed by Ministry of Human Resource Development. Mid Day Meal Scheme was launched by Ministry of Human Resource Development with effect from 15th August 1995. Foodgrains are supplied free of cast @ 100gram per child per school day where cooked meal is being seved with minimum content of 300 calories and 8-12 gram protein each day of school for minimum of 200 days.

124. In which of the following Committees there is no representation of Rajya Sabha?
(a) Public Accounts Committee
(b) Committee on Public Undertakings
(c) Estimates Committee
(d) Committee on Government Assurances
Ans. c
Explanation:
While members of the Rajya Sabha are associated with Committee on Public Accounts and Public Undertakings, the mebmers of the Committee on Estimates are drawn entirely from the Lok Sabha.

125. Provision regarding Panchayats and Municipalities was made in the Indian Constitution in which year?
(a) 1991 (b) 1995
(c) 2000 (d) 1993
Ans. d
Explanation:
The 73rd Amendment Act of 1992 gave constituttional recognition to the panchayats by adding a new Part IX and new Schedule 11 to the Constitution. Similarly, 74rd Amendment Act of 1992 gave constituttional recognition to the municipalities by adding a new Part IX-A and new Schedule 112 to the Constitution. Both Amendment Acts came inti force in 1993.

126. The Provision for the Calling Attention Notices has restricted the scope of which of the following?
(a) Short duration discussion
(b) Question hour
(c) Adjournment motion
(d) Zero hour
Ans. a
Explanation:
The Provision for the Calling Attention Notices has restricted the scope of Short duration discussion. Calling Attention Notice is introduced in the Parliament by a member to the call the attention of the minister to a matter of urgent public importance, and seek authoritativre statement from him on that matter. In Short duration discussion, the Parliament members can raise such discussion on a matter of urgent public importance.

127. Match List-I with List-II and select the correct answer from the codes given below the lists
List-I List-II
A. Article 14 1. Amendment
Procedure
B. Article 36 2. Council of
Ministers
C. Article 74 3. Right to Equality
D. Article 368 4. Directive Principles
Codes:
A B C C
(a) 2 1 4 3
(b) 4 1 3 2
(c)1 2 3 4
(d) 3 4 2 1
Ans. d
Explanation:
Article 14- Right to Equality
Article 36- Directive Principles
Article 74- Council of Ministers
Article 368- Amendment Procedure

128. The following States were created after 1960. Arrange them in ascending chronological order of their formation and choose your answer from the given code.
1. Haryana 2. Sikkim
3. Nagaland 4. Meghalaya
Codes:
(a) 1,2,3,4 (b) 2,3,4,1
(c) 3,1,4,2 (d) 2,4,1,3
Ans. c
Explanation:
Nagaland-1963
Haryana-1966
Meghalaya-1972
Sikkim-1974

129. The function of the Pro-Temp. Speaker is to
(a) conduct the proceedings of the House in the absence of Speaker.
(b) swear in members.
(c) officiate as Speaker when the Speaker is unlikely to be elected.
(d) only check if the election certificates of the members are in order.
Ans. c (most correct answer)
Explanation:
The president appoints and administers oath to the Speaker Pro Tem. Speaker Pro Tem presides over the first sitting of the newly elected Lok Sabha and administers oath to the new members. He also enables the House to elect the new spesker.

130. Reema is twice as old as Sumita. Three years ago the was three times as old as Sumita. How old is Reema now?
(a) 6 years (b) 7 years
(c) 8 eyars (d) 12 years
Ans. d
Explanation:
Let
Age of Reema=R
Age of Sumita=S
Reema is twice as old as Sumita
R=2XS---------------(1)
Three years ago the was three times as old as Sumita
R-3=(S-3)X3---------(2)
Replacing value of R in equation number (2)
2S-3=3S-9
S=6
R=6X2=12 years

131. The marked price of a table is Rs. 2,000, which is 25% above the cost price. If the shopkeeper sells the table at 8% discount on the marked price, his percentage of profit would be
(a) 14% (b) 15%
(c) 16% (d) 20%
Ans. b
Explanation:
Let
Cost price= X
X + Xx25/100=2,000
X=1,600
Selling price=2,000-2,000x8/100=1,840
Profit=240x100/1600=15%

132. Which of the following statements is not true of the Indian Economy?
(a) Its share of world population is only 16% but its share of world GDP is 1.6%.
(b) The share of service sector in India's GDP is only 25%
(c) 58% of its working population is engaged in agriculture, but the contribution of agriculture to the national income is 22%
(d) India occupies only 2.4% of the world's geographical area.
Ans. b
Explanation:
Share of agriculture and allied sectors in GDP at factor cast has declined gradually to 14.6 % in 2009-10. During same period , the share of industry has remained the same at about 28 %, while that of services has gone up to 57.2 %.

133. The expenses of Public Service Commission of Uttar Pradesh are charged upon
(a) Conolidated Fund of India
(b) Cosolidated Fund of State
(c) It's own generated fund from fees
(d) Contringent Fund
Ans. b
Explanation:
The expenses of Public Service Commission of Uttar Pradesh are charged upon cosolidated Fund of State. Thus, they are not subjucted to vote of the state legislature.

134. In 1930 a person's age was 8 times that of his son. In 1938 the father's age becomes 10 times the age of his son's age in 1930. The ages of father and son in 1940 were respectively
(a) 58 and 16 years
(b) 50 and 15 years
(c) 42 and 14 years
(d) 34 and 13 years
Ans. c
Explanation:
Let
Father’s age in 1930 =F
Son’s age in 1930=S
In 1930 a person's age was 8 times that of his son.
F=8XS---------------------------(I)
In 1938 the father's age becomes 10 times the age of his son's age in 1930.
F+8=10(S)-----------------------(II)
Replacing value of F in equation number (II)
8XS+8=10XS
2S=8
S=4
F=32
The ages of father and son in 1940 were respectively
Father’s age=32+10=42
Son’s age =4+10= 14

135. Who is the brand ambassador of BSNL?
(a) Shah Rukh Khan
(b) Abhishek Bachchan
(c) Priety Zinta
(d) Deepika Padukone
Ans. d
Explanation:
Actress Deepika Padukone has been appointed by Bharat Sanchar Nigam Ltd (BSNL) as its new brand ambassador. She replaces Preity Zinta, who was until now endorsing the brand.

136. The headquarer of R.B.I. is situated at
(a) Delhi
(b) Kolkata
(c) Mumbai
(d) Chennai
Ans. c
Explanation:
The Reserve Bank of India was established on April 1, 1935 in accordance with the provisions of the Reserve Bank of India Act, 1934.
The Central Office of the Reserve Bank was initially established in Calcutta but was permanently moved to Mumbai in 1937. Though originally privately owned, since nationalisation in 1949, the Reserve Bank is fully owned by the Government of India.
137. An ox in a 'Kolhu' is tethered to a rope 3 m long. How much distance does the ox cover is 14 rounds?
(a) 300 m (b) 250 m
(c) 264 m (d) 232 m
Ans. c
Explanation:
Circumference=2πr
Distance covered in 14 rounds=2πrX14
=2x22/7x3x14=264


138. A can finish a work in 6 days whereas B can finish the work in 9 days. If both of them work together, what is the probability that the work will be finished on third day?
(a) ½ (b) 1/6
(c) 1 (d) 0
Ans. d (?)
Explanation:
A can finish a work in 6 days.
A’s 1 day’s work= 1/6
Whereas B can finish the work in 9 days.
B’s 1 day’s work-1/9
If both work togather
1 day’s work= 1/6 +1/9= 10/36
If both work togather
Work will be copleted= 36/10=3.6 days

139. Which layer of atmosphere is responsible for Aurora Borealis
(a) Troposphere
(b) Thermosphere
(c) Ionospehre
(d) Exosphere
Ans. c
Explanation:
The aurora borealis is also called the northern polar lights, as it is only visible in the sky from the Northern Hemisphere. Earth's magnetosphere is formed by the impact of the solar wind on the Earth's magnetic field. It forms an obstacle to the solar wind, diverting it. Auroras have also been observed on Venus and Mars.

140. The Ruhr basin is the famous Industrial region of
(a) China
(b) Germany
(c) Japan
(d) United Kingdom
Ans. b
Explanation:
Ruhr river is situated in WC Germany, flowing west into the Rhine and major coal-mining & industrial region centered in the valley of this river is called Ruhr Basin

141. Which of the following countries are located on the Equator?
1. Brunei 2. Columbia
3. Kenya 4. Venezuela
Select the correct answer using the codes given below:
Coces:
(a) 1 and 2 (b) 2 and 3
(c) 3 and 4 (d) 1 and 4
Ans. b
Explanation: Countries that Lie on the Equator
1. Sao Tome and Principe
2. Gabon
3. Republic of The Congo
4. Democratic Republic of The Congo
5. Uganda
6. Kenya
7. Somalia
8. Indonesia
9. Kiribati (the equator may or may not touch dry land)
10. Ecuador
11. Colombia
12. Brazil

142. If the Sun rises at RIRAP in Arunachal Pradesh at 5.00 am (IST), then what time (IST) the Sun will rise in Kandla in Gujarat?
(a) About 5.30 a.m.
(b) About 6.00 a.m.
(c) About 7.00 a.m.
(d) About 7.30 a.m.
Ans. c
Explanation:
At present, India’s time is set in accordance with the 82.5 degrees East longitude that determines the Indian Standard Time (IST), set five-and-a-half-hours ahead of Greenwich Mean Time.
If the sun rises at 5.00 AM at Tirap in Arunachal, it is only after about two hours that Kandla in Gujarat will get to see sunrise.

143. Which one of the following islands of the East Indies is divided into three countries?
(a) Borneo (b) Celebes
(c) new Guinea (d) Timor
Ans. a
Explanation:
Borneo is the third largest island in the world, located north of Australia, at the geographic centre of Maritime Southeast Asia. Politically, the island is divided among three countries: Indonesia, Malaysia and Brunei. Approximately 73% of the island is Indonesian territory; the Indonesian named for the island, Kalimantan.

144. 900E Ridge lies in
(a) Atlantic Ocean
(b) Indian ocean
(c) Pacific ocean
(d) Mediterranean sea
Ans. b
Explanation:
90 degree east ridge is an aseismic ridge it is so called due to the lack of seismicity. The volcanics from the Ninety East Ridge in the Indian Ocean consist of basalts and oceanic andesites.

145. Which of the following rivers flow through a 'rift' valley?
(a) Ganga
(b) Brahamputra
(c) Narmada
(d) Krishna
Ans. c
Explanation:
The Narmada is one of only three major rivers in peninsular India that runs from east to west (largest west flowing river) along with the Tapti River and the Mahi River. It is the only river in India that flows in a rift valley flowing west between the Satpura and Vindhya ranges although the Tapti River and Mahi River also flow through rift valleys but between different ranges.

146. Which soil needs little irrigation as it retains soil moisture?
(a) Alluvial soil
(b) Black soil
(c) Red soil
(d) Laterite soil
Ans. b
Explanation:
Black soil needs little irrigation as it retains soil moisture. Among the in situ soils of India, the black soils found in the lava-covered areas are the most conspicuous. These soils are often referred to as regur but are popularly known as “black cotton soils,” since cotton has been the most common traditional crop in areas where they are found.

147. In an area with annual rainfall of more than 200 cms and sloping hills which crop will be ideal?
(a) Jute (b) Cotton
(c) Tea (d) Maize
Ans. c
Explanation:
In an area with annual rainfall of more than 200 cms and sloping hills is ideal for tea cultivation.

148. Match List-I with List-II and select the correct answer from the codes given below the lists
List-I List-II
A. Etna 1. Ross Island
B. Vesuvius 2. Ecuador
C. Erebus 3. Italy
D. Cotopaxy 4. Sicily
Codes:
A B C D
(a) 1 2 3 4
(b) 4 3 1 2
(c) 3 4 2 1
(d) 4 3 2 1
Ans. b
Explanation:
Cotopaxy- Ecuador
Vesuvius- Italy
Mount Erebus in Antarctica is the southernmost historically active volcano on Earth. With a summit elevation of 3,794 metres (12,448 ft), it is located on Ross Island, which is also home to three inactive volcanoes, notably Mount Terror. Mount Erebus is part of the Pacific Ring of Fire, which includes over 160 active volcanoes.

149. The Anglo-American Culture Realm does not include
1. Canada 2. U.S.A.
3. Maxico 4. Cuba
Select the correct answer from the codes given below:
Codes:
(a) 1 and 2 (b) 1 and 3
(c) 2 an 3 (d) 3 and 4
Ans. d
Explanation:
Pertaining to Anglo-America, a term denoting an area of mixed English and American influence or heritage, or those parts of or groups within the Americas which have a tie to or which are influenced by England; or simply English-speaking America.

150. The Kalpsar Project for supply of sweet water is located in
(a) Gujarat
(b) Haryana
(c) Maharashtra
(d) Rajasthan
Ans. a
Explanation:
A project thus conceived initially as a tidal power project was transformed as tidal power cum fresh water supply project providing large multipurpose benefits. It is in fact a project envisaging total multipurpose development of the Gulf of Khambha
read more...

IAS 2010 PRE GS Exam paper with Solution

1. For the Karachi session of Indian National Congress in 1931 presided over by Sardar Patel, who drafted the Resolution on Fundamental Rights and Economic Programme?
(a) Mahatma Gandhi
(b) Pandit Jawaharlal Nehru
(c) Dr. Rajendra Prasad
(d) Dr. B.R. Ambedkar
Ans. b
Explanation:
In 1931, the Indian National Congress, at its Karachi session, adopted resolutions defining, as well as committing itself to the defence of fundamental civil rights, including socio-economic rights such as minimum wage, the abolition of untouchability and serfdom. Pandit Jawaharlal Nehru drafted the Resolution on Fundamental Rights and Economic Programme.

2. Who among the following were official Congress negotiators with Cripps Mission?
(a) Mahatma Gandhi and Sardar Patel
(b) Acharya J.B. Kripalani and C. Rajagopalachari
(c) Pandit Nehru and Maulana Azad
(d) Dr. Rajendra Prasad and Rafi Ahmed Kidwai
Ans. c
Explanation:
Pandit Nehru and Maulana Azad were official Congress negotiators with Cripps Mission. The Cripps mission was an attempt in late March 1942 by the British government to secure Indian cooperation and support for their efforts in World War II.

3. Which one of the following processes in the bodies of living organisms is a digestive process?
(a) Breakdown of proteins into amino acids
(b) Breakdown of glucose into CO2 and H2O
(c) Conversion of glucose into glycogen
(d) Conversion of amino acids into proteins
Ans. a
Explanation:
Digestion is the mechanical and chemical breaking down of food into smaller components that can be absorbed into a blood stream, for instance. eg. Break-down of carbohydrates, proteins and fats into constituent units.

4. From the point of view of evolution of living organisms, which one of the following is the correct sequence of evolution?
(a) Otter - Tortoise - Shark
(b) Shark - Tortoise - Otter
(c) Tortoise - Shark - Otter
(d) Shark - Otter – Tortoise
Ans. b
Explanation:
Otters are semi-aquatic (or in one case aquatic) fish-eating mammals.

5. Consider the following statements:
1. Hepatitis B is several times more infectious than HIV/AIDS
2. Hepatitis B can cause liver cancer Which of the statements given above is/are correct?
(a) 1 only
(b) 2 only
(c) Both 1 and 2
(d) Neither 1 nor 2
Ans. c
Explanation:
Hepatitis B is several times more infectious than HIV/AIDS because HIV is heat sensitive.
Chronic hepatitis B may eventually cause liver cirrhosis and liver cancer—a fatal disease with very poor response to current chemotherapy. The infection is preventable by vaccination.

6. Excessive release of the pollutant carbon monoxide (CO) into the air may produce a condition in which oxygen supply in the human body decreases. What causes this condition?
(a) When inhaled into the human body, CO is converted into CO2
(b) The inhaled CO has much higher affirnity for haemoglobin as compared to oxygen
(c) The inhaled CO destroys the chemical structure of haemoglobin
(d) The inhaled CO adversely affects the respiratory centre in the brain.
Ans. b
Explanation:
The inhaled CO has much higher affirnity for haemoglobin as compared to oxygen. Carbon monoxide mainly causes adverse effects in humans by combining with hemoglobin to form carboxyhemoglobin (HbCO) in the blood. This prevents oxygen binding to hemoglobin, reducing the oxygen-carrying capacity of the blood, leading to hypoxia.

7.. Consider the following statements:
1. Every individual in the population is equally susceptible host for Swine Flue.
2. Antibiotics have no role in the primary treatment of Swine Flu.
3. To prevent the future spread of Swine Flu in the epidemic area, the swine (pigs) must all be culled.
Which of the statements given above is/are correct?
(a) 1 and 2 only
(b) 2 only
(c) 2 and 3 only
(d) 1, 2 and 3
Ans. b
Explanation:
People who work with poultry and swine, especially people with intense exposures, are at increased risk of zoonotic (bird and swine flue) infection. People in at-risk groups should be treated with antivirals (oseltamivir or zanamivir) as soon as possible when they first experience flu symptoms. Antibiotics have no role in the primary treatment of Swine Flu. The World Health Organization moved to put the pandemic alert to phase 5 confirms that the situation is not a pig problem but a human problem.

8. With regard to the transmission of the Human Immunodeficiency Virus, which one of the following statements is not correct?
(a) The chances of transmission from female to male are twice as likely as from male to female
(b) The chances of transmission are more if a person suffers from other sexually transmitted infections
(c) An infected mother can transmit the infection to her baby during pregnancy, at childbirth and by breast feeding
(d) The risk of contracting infection from transfusion of infected blood is much higher than an exposure to contaminated needle
Ans. a
Explanation:
In high-income countries, the risk of female-to-male transmission is 0.04% per act and male-to-female transmission is 0.08% per act. The transmission of the virus from the mother to the child can occur in utero during the last weeks of pregnancy and at childbirth. Breastfeeding also increases the risk of transmission by about 4 %.

9. What are the possible limitations of India in mitigating the global warming at present and in the immediate future?
1. Appropriate alternate technologies are not sufficiently available.
2. Indian cannot invest huge funds in research and development.
3. Many developed countries have already set up their polluting industries in India.
Which of the statements given above is/are correct?
(a) 1 and 2 only
(b) 2 only
(c) 1 and 3 only
(d) 1, 2 and 3
Ans. a
Explanation:
No need of explanation.

10. Consider the following statements:
1. The Commonwealth has no charter treaty or constitution.
2. All the territories/countries once under the British empire (jurisdiction/rule/mandate) automatically joined the Commonwealth as its members.
Which of the statements given above is/are correct?
(a) 1 only
(b) 2 only
(c) Both 1 and 2
(d) Neither 1 nor 2
Ans. a (?)
Explanation: Membership is purely voluntary. A number of territories formerly under British jurisdiction or mandate did not join the Commonwealth like Egypt, Iraq, Myanmar etc..

11. Examine the following statements:
1. All colours are pleasant.
2. Some colours are pleasent.
3. No colour is pleasant.
4. Some colours are not pleasant.
Given that the statement 4 is true, what can be definitely concluded?
(a) 1 and 2 are true
(b) 1 is false
(c) 2 is false
(d) 3 is true
Ans. b

12. In the context of bilateral trade negotiations between Indian and European Union, what is the difference between European Commission and European Council?
1. European Commission represents the EU in trade negotiations whereas European Council participates in the legislation of matters pertaining to economic polices of the European Union.
2. European Commission comprises the Heats of State or government of member countries whereas the European Council comprises of the persons nominated by European Parliament.
Which of the statements given above is/are correct?
(a) 1 only
(b) 2 only
(c) Both 1 and 2
(d) Neither 1 nor 2
Ans. d
Explanation:
The European Council is the institution of the European Union (EU) responsible for defining the general political direction and priorities of the Union. It comprises the heads of state or government of EU member states, along with its President and the President of the Commission. The High Representative takes part in its meetings, which are chaired by its President: currently Herman Van Rompuy.
While the European Council has no formal legislative power, it is an institution that deals with major issues and any decisions made are "a major impetus in defining the general political guidelines of the European Union".

13. The approximate representation of land use classification in India is
(a) Net area sown 25%; forests 33%; other areas 42%
(b) Net area sown 58%; forests 17%; other areas 25%
(c) Net area sown 43%; forests 29%; other areas 28%
(d) Net area sown 47%; forests 23%; other areas 30%
Ans. d (most correct answer)
Explanation: India 2010 page-308
According to State of Forest report 2005, the total forest cover of the country as per 2005 assessment is 20.65 of total geographical area of country.

14. With reference to the National Investment Fund to which the disinvestment proceeds are routed, consider the following statements:
1. The assets in the National Investment Fund are managed by the Union Ministry of Finance.
2. The National Investment Fund is to be maintained within the Consolidated Fund of India.
3. Certain Asset Management Companies are appointed as the fund managers.
4. A certain proportion of annual income is used for financing select social sectors.
Which of the statements given above is/are correct?
(a) 1 and 2
(b) 2 only
(c) 3 and 4
(d) 3 only
Ans. c
Explanation:
NIF would be maintained outside the Consolidated Fund of India and would be professionally managed by selected public sector mutual funds to provide sustainable return without depleting the corpus. 75% of annual income of NIF will be used to finance selected social sector schemes. The three AMCs (the UTI Asset Management Company Pvt. Ltd., the SBI Funds Management Pvt. Ltd. and the LIC Mutual Fund Asset Management Company Ltd.) have been appointed as fund managers for NIF, initially for two years, but extendable later on the basis of their performance. Separate agreements were signed with each fund manager.

15. In India, which of the following is regulated by the Forward Markets Commission?
(a) Currency Futures Trading
(b) Commodities Futures Trading
(c) Equity Futures Trading
(d) Both Commodities Futures and Financial Futures Trading.
Ans. b
Explanation:
The Forward Markets Commission (FMC) is the chief regulator of forwards and futures markets in India. It regulates commodity trade in India. It is headquartered in Mumbai and is overseen by the Ministry of Consumer Affairs, Food and Public Distribution, Government of India.

16. Which one of the following is not a feature of Limited Liability Partnership firm?
(a) Partners should be less than 20
(b) Partnership and management need not be separate
(c) Internal governance may be decided by mutual agreement among partners.
(d) It is corporate body with perpetual succession
Ans. a
Explanation:
LLP shall be a body corporate and a legal entity separate from its partners. It will have perpetual succession. Indian Partnership Act, 1932 shall not be applicable to LLPs and there shall not be any upper limit on number of partners in an LLP unlike an ordinary partnership firm where the maximum number of partners can not exceed 20, LLP Act makes a mandatory statement where one of the partner to the LLP should be an Indian. A minimum of two partners will be required for formation of an LLP.

17. With reference to the institution of Banking Ombudsman in India, which one of the statements is not correct?
(a) The Banking Ombudsman is appointed by the Reserve Bank of India
(b) The Banking Ombudsman can consider complaints from Non-Resident Indians having accounts in india
(c) The orders passed by the Banking Ombudsman are final and binding on the parties concerned
(d) The service provided by the Banking Ombudsman is free of any fee
Ans. c
Explanation:
The Banking Ombudsman is a senior official appointed by the Reserve Bank of India to redress customer complaints against deficiency in certain banking services.
The Banking Ombudsman can receive and consider any complaint relating to complaints from Non-Resident Indians having accounts in India in relation to their remittances from abroad, deposits and other bank-related matters; If one is not satisfied with the decision passed by the Banking Ombudsman, one can approach the appellate authority against the Banking Ombudsmen’s decision. Appellate Authority is vested with a Deputy Governor of the RBI.

18. With reference to India, consider the following:
1. Nationalization of Banks
2. Formation of Regional Rural Banks
3. Adoption of villages by Bank Branches
Which of the above can be considered as steps taken to achieve the "financial inclusion" in India?
(a) 1 and 2 only
(b) 2 and 3 only
(c) 3 only
(d) 1, 2 and 3
Ans. d
Explanation: Steps taken to achieve the "financial inclusion" in India-
a. A hugely expanded bank branch and cooperative network and new organizational forms like RRBs;
b. A greater focus on credit rather than other financial services like savings and insurance, although the banks and cooperatives did provide deposit facilities;
c. Nationalization of Banks

19. What was the immediate reason for Ahmad Shah Abdali to invade India and fight the Third Battle of Panipat?
(a) He wanted to avenge the expulsion by marathas of his viceroy Timur "Shah from Lahore
(b) The frustrated governor of Jullundhar Adina Beg Khan invited him to invade Punjab
(c) he wanted to punish Mughal administration for non-payment of the revenues of the Chahar Mahal (Gujarat, Aurangabad, Sialkot and Pasrur)
(d) He wanted to annex all the fertile plains of Punjab up the borders of Delhi to his kingdom
Ans. a
Explanation:
To counter the Afghans, Peshwa Balaji Bajirao sent Raghunathrao. He succeeded in ousting Timur Shah and his court from India and brought Lahore, Multan, Kashmir and other subahs on the Indian side of Attock under Maratha rule. Thus, upon his return to Kandahar in 1757, Amidst appeals from Muslim leaders like Shah Waliullah, Ahmad Shah chose to return to India and confront the Maratha Confederacy.

20. With reference to Pondicherry (now Pundchery), consider the following statements:
1. The first European power to occupy Pondicherry were the Portuguese.
2. The second European power to occupy Pondicherry were the French.
3. The English never occupied Pondicherry.
Which of the statements given above is/are correct?
(a) 1 only
(b) 2 and 3 only
(c) 3 only
(d) 1, 2 and 3
Ans. c (?)
Explanation:
Joseph François Dupleix became the Governor of the French Territory in India on 15 January 1742 and brought Madras also under French control in September 1746; Madras continued under French rule for 30 years. Internal disturbances in Pondicherry gave the British the opportunity, in August 1793, to gain control of Pondicherry; it was administered as part of Madras till 1815. However, after the Treaty of Paris in 1814, the British restored the settlements, which the French had possessed on 1 January 1792, back to the French in 1816. French rule continued till 31 October 1954.

21. Why did Buddhism start declining in India in the early medieval times.
1. Buddha was by that time considered as one of the incarnations of Vishnu and thus became a part of Vaishnavism.
2. The invading tribes from Central Asia till the time of last Gupta king adopted Hinduism and presecuted Buddhists.
3. The Kings of Gupta dynasty were strongly opposed to Buddhism.
Which of the statements given above is/are correct?
(a) 1 only
(b) 1 and 3 only
(c) 2 and 3 only
(d) 1, 2 and 3
Ans. a
Explanation:
The successor Shungas reinstated the sacrifices and persecuted Buddhism. Central Asian and North Western Indian Buddhism weakened in the 6th century following the White Hun invasion, who followed their own religions. The Gupta Empire period was a time of great development of Hindu culture, but even then in the Ganges Plain half of the population supported Buddhism.

22. Consider the following statements: The functions of commercial banks in India include
1. Purchase and sale of shares and securities on behalf of customers.
2. Acting as executors and trustees of wills.
Which of the statement given above is/are correct?
(a) 1 only
(b) 2 only
(c) Both 1 and 2
(d) Neither 1 nor 2
Ans. d
Explanation:
The modern Commercial Banks in India cater to the financial needs of different sectors. The main functions of the commercial banks comprise:
• transfer of funds
• acceptance of deposits
• offering those deposits as loans for the establishment of industries
• purchase of houses, equipments, capital investment purposes etc.
• The banks are allowed to act as trustees. On account of the knowledge of the financial market of India the financial companies are attracted towards them to act as trustees to take the responsibility of the security for the financial instrument like a debenture.
• The Indian Government presently hires the commercial banks for various purposes like tax collection and refunds, payment of pensions etc.
23. In India, the tax proceeds of which one of the following as a percentage of gross tax revenue has significantly declined in the last five years?
(a) Service tax
(b) personal income tax
(c) Excise duty
(d) Corporation tax
Ans. c
Explanation: Economic Survey 2009-10 page-42

24. Which one of the following authorities makes recommendation to the Governor of a State as to the principles for determining the taxes and duties which may be appropriated by the Panchayats in that particular State?
(a) District Planning Committees
(b) State Finance Commission
(c) Finance Ministry of that State
(d) Panchayati Raj Ministry of that State
Ans. b
Explanation:
Governor of a State shall after every 5 years constitute a finance commission to review the financial position of Panchayats in that particular State. State Finance Commission makes recommendation to the Governor of a State as to the principles for determining the taxes and duties which may be appropriated by the Panchayats in that particular State.

25. Consider the following statements: In India, taxes on transactions in Stock Exchanges and Futures Markets are
1. levied by the Union
2. collected by the States
Which of the statements given above is/are correct?
(a) 1 only
(b) 2 only
(c) Both 1 and 2
(d) Neither 1 nor 2
Ans.
Explanation:
Some taxes are levied and collected by the Centre but the proceeds are to be distributed among States. These include succession and estate duties in respect of property other than agricultural land, terminal tax on goods and passengers, tax on railway fares and freights, taxes on transaction in stock exchanges and future markets, taxes on sale or purchase of newspapers and ads.

26. In India, during the last decade the total cultivated land for which one of the following crops has remained more or less stagnant?
(a) Rice
(b) Oilseeds
(c) Pulses
(d) Sugarcane
Ans. a (Economic Survey 2009-10 page-182)
Explanation:
In India, during the last decade the total cultivated land for rice has remained more or less stagnant.

27. Consider the following statements:
1. The Union Government fixes the Statutory Minimum Price of sugarcane for each sugar season.
2. Sugar and sugarcane are essential commodities under the Essential Commodities Act.
Which of the statements given above is/are correct?
(a) 1 only
(b) 2 only
(c) Both 1 and 2
(d) Neither 1 nor 2
Ans. b
Explanation:
The control on sugar is effected mainly through control orders issued under the Essential Commodities Act and some State legislation, like the one in Uttar Pradesh.
Central Government fixes the Statutory Minimum Price (SMP) of sugarcane for each sugar season under the Sugarcane (Control) Order, 1966 for each sugar factory. SMP is fixed on the basis of the recommendations of the commission on Agricultural Costs & Prices (CACP ).Some State Governments announce State Advised Prices (SAP) for sugarcane for their sugar factories. SAPs are higher than SMP. SAPs are not fixed on any scientific basis. SAPs are not statutorily binding.
SMP is now replaced or supplemented by Fair and Reasonable Price (FRP) and State Advised Price (SAP).

28. With reference to Indian economy consider the following statements:
1. The Gross Domestic Product (GDP) has increased by four times in the last 10 years
2. The percentage share of Public Sector in GDP has declined in the last 10 years.
Which of the statements given above is/are correct?
(a) 1 only
(b) 2 only
(c) Both 1 and 2
(d) Neither 1 nor 2
Ans. d (?)
Explanation:
The Gross Domestic Product (GDP) has increased by three times in the last 10 years.

29. Consider the following which can be found in the ambient atmosphere:
1. Soot
2. Sulphur hexafluoride
3. Water vapour
Which of the above contribute to the warming up of the atmosphere?
(a) 1 and 2 only
(b) 3 only
(c) 2 and 3 only
(d) 1, 2 and 3
Ans. d
Explanation:
The main greenhouse gases in the Earth's atmosphere are water vapor, carbon dioxide, methane, nitrous oxide, and ozone. According to the Intergovernmental Panel on Climate Change, SF6 is the most potent greenhouse gas that it has evaluated, with a global warming potential of 22,800[5] times that of CO2 when compared over a 100 year period. CO2 is resposible for 40% of earth warming and black carbon for 12%.

30. The International Development Association, a lending agency, is administrated by the
(a) International Bank for Reconstruction and Development
(b) International Fund for Agricultural Development
(c) United Nations Development Programme.
(d) United Nations Industrial Development Organization.
Ans. a
Explanation:
The IDA is administered by the same officers and staff who administer the affairs of the IBRD. The president of the Bank also serves as the president of the IDA, and the governors and the executive directors of the Bank serve in the same capacity in the IDA. As in the IBRD, a member's voting power in the IDA is roughly proportionate to its capital subscription.

31. When you travel in certain parts of India, you will notice red soil. What is the main reason for this colour?
(a) Abundance of magnesium
(b) Accumulated humus
(c) Presence of ferric oxides
(d) Abundance of phosphates
Ans. c
Explanation:
Red soil is formed as a result of the wearing down of the old crystalline rocks. It is less clayey and more of a sandy nature and has a rich content of iron and small Humus content. Red soil is poor in nitrogen, phosphorus and lime. Reddish color of red soil is due to wide diffusion of iron.

32. Which one of the following is the appropriate reason for considering the Gondana rocks as most important of rock systems of India?
(a) More than 90% of limestone reserves of India are found in then
(b) More than 90% of India's coal reserves are found in them
(c) More than 90% of fertile black cotton soils are spread over them
(d) None of the reasons given above is appropriate in this context
Ans. b
Explanation:
Gondwana rocks are economically most important rock system in India containing about 98% of her coal reserve.

33. Which one of the following can one come across is one travels through the Strait of Malacca?
(a) bali
(b) Brunei
(c) Java
(d) Singapore
Ans. d
Explanation:
See any Atlas.

34. With reference to the river Luni, which one of the following statements is correct?
(a) It flows into Gulf of Khambhat
(b) it flows into Gulp of Kuchhh
(c) It flows into pakistan and mergers with a tributary of Indus
(d) It is lost in the marshy land of the Rann of Kuchchh
Ans. d
Explanation:
The Luni is a river of western Rajasthan state, India. It originates in the Pushkar valley of the Aravalli Range, near Ajmer and ends in the marshy lands of Rann of Kutch in Gujarat

35. Which one of the following pairs in not correctly matched?
Dam/Lake River
(a) Govind Sagar Satluj
(b) Kolleru lake Krishna
(c) Ukai Reservoir Tapi
(d) Wular Lake Jhelum
Ans. b
Explanation:
Govind Sagar- Gobind Sagar is a man-made reservoir situated in Bilaspur District, Himachal Pradesh.The reservoir on the river Sutlej, was formed after the hydel dam at Bhakra was constructed and has been named in honour of Guru Gobind Singh. Kolleru lake- Kolleru Lake is the largest freshwater lake in India. It is located in Andhra Pradesh state, India. Kolleru is located between Krishna and Godavari delta. Ukai Reservoir- Ukai Dam, constructed across the Tapti River, is the largest reservoir in Gujarat. Wular Lake- Wular Lake is a large fresh water lake in Bandipore district in the Indian state of Jammu and Kashmir. The lake basin was formed as a result of tectonic activity and is fed by the Jhelum River.
36. A geographic region has the following distinct characteristics:
1. Warm and dry climate
2. Mild and wet winter
3. Evergreen oak trees
The above features are the distinct characteristics of which one of the following regions?
(a) Mediterranean
(b) Eastern China
(c) Central Asia
(d) Atlantic coast of North America
Ans. a
Explanation:
Mediterranean region is characterized by Warm and dry climate, Mild and wet winter and Evergreen oak trees.

37. Which reference to the National Rehabilitation and Resettlement Policy, 2007, consider the following statements:
1. This policy is applicable only to the persons affected by the acquisition of land for projects and not to the involuntary displacement due to any other reason.
2. This policy has been formulated by the Ministry of Social Justice and Empowerment.
Which of the statements given above is/are correct?
(a) 1 only
(b) 2 only
(c) Both 1 and 2
(d) Neither 1 nor 2
Ans. d
Explanation:
The new Bill provides for benefits and compensation to people displaced by land acquisition purchases or any other involuntary displacement. For large scale displacement, the government shall conduct a social impact assessment, and appoint an Administrator for Rehabilitation and Resettlement to formulate and execute the rehabilitation and resettlement plan. This policy has been formulated by the Ministry of Rural Development.

38. In the context of India's Five Year Plans, a shift in the pattern of industrialization, with lower emphasis on heavy industries and more on infrastructure begins in
(a) Forth Plan
(b) Sixth Plan
(c) Eighth Plan
(d) Tenth Plan
Ans. b (India 2010 page-809)
Explanation:
Strategy adapted during sixth plan was to move simultaneously toward strengthening the infrastructure for both agriculture and industries.

39. Two of the schemes launched by the Government of India for Women's development are Swadhar and Swayam Siddha. As regards the difference between them, consider the following statements:
1. Swayam Siddha is meant for those in difficult cicumstances such as women survivors of natural disasters or terrorism, women prisoners released from jails, mentally challenged women etc. whereas Swadhar is meant for holistic empowerment of women through Self Help Groups.
2. Swayam Siddha is implemented through Local Self Government bodies or reputed Vountary Organizations whereas Swadhar is implemented through the ICDS units set up in the states.
Which of the statements given above is/are correct?
(a) 1 only
(b) 2 only
(c) Both 1 and 2
(d) Neither 1 nor 2
Ans. d (India 2010 page-1059,60)
Explanation:
Swayam Sidha is based on formation of women into self help groups and programme is being implemented by ICDS machinery in most of States. The Swadhar Scheme purports to address the specific vulnerability of each of group of women in difficult circumstances through a Home-based holistic and integrated approach.

40. With reference to the United Nations Convention on the Rights of the Child, consider the following:
1. The Right to Development
2. The Right to Expression
3. The Right to Recreation
Which of the above is/are the Rights of the child?
(a) 1 only
(b) 1 and 3 only
(c) 2 and 3 only
(d) 1, 2 and 3
Ans. d
Explanation:
The United Nations Convention on the Rights of the Child is an international statement of the civil, political, economic, social and cultural rights of children.

41. Three men start together to travel the same way around a circular track of 11 km. Their speeds are 2, 5.5 and 8 kmph respectively. When will they meet at the starting point for the first time?
(a) After 11 hours
(b) After 21 hours
(c) After 22 hours
(d) After 33 hours
Ans. d
Explanation:
Circumference
= 2πr
= 69.14
Time taken by first person
=69.14/4
=17.2 hours
Time taken by second person
=69.14/5.5
= 12.57 hours
Answer should be more than 34.4 hours

42. With reference to the Consumer Disputes Redressal at district level in India, which one of the following statements is not correct?
(a) A State Government can establish more than one District Forum in a district if it deems fit
(b) One of the members of the District Forum shall be a women
(c) The District Forum entertains the complaints where the value of goods or services does not exceed rupees fifty laksh
(d) A complaint in relation to any goods sold or any service provided may be filed with a District Forum by the State Government as a representative of the interests of the consumers in general
Ans. c
Explanation:
Consumer Disputes Redressal Forum to be known as the "District Forum" established by the State Government in each district of the State by notification: Provided that the State Government may, if it deems fit, establish more than one District Forum in a district. Each District Forum shall consist of,— a person who is, or has been, or is qualified to be a District Judge, who shall be its President; two other members, one of whom shall be a woman. Jurisdiction of the District Forum.—(1) Subject to the other provisions of this Act, the District Forum shall have jurisdiction to entertain complaints where the value of the goods or services and the compensation, if any, claimed ''does not exceed rupees twenty lakhs.

43. King Cobra is the only snake that makes its own nest. Why does it make its nest?
(a) It is snake-eater and the nest helps attract other snakes
(b) It is a viviparous snake and needs a nest to given birth to its offspring
(c) It is a viviparous snake and lays its eggs in the nest and guards the nest until they are hatched
(d) It is a large, cold blooded animal and needs a nest to hibernate in the cold season
Ans. c
Explanation:
Before she is ready to lay her eggs, she uses the coils of her long body to gather a big mound of leaf litter. She deposits 20–40 eggs into the mound, which acts as an incubator. The female stays with her eggs and guards the mound tenaciously, rearing up into a threat display if any large animal gets too close. When the eggs start to hatch, instinct causes her to leave the nest and find prey to eat so that she does not eat her young.

44. As a result of their annual survey, the National Geographic Society and an international polling firm Gobe Scan gave India top rank in Greendex 200 score. What is this score?
(a) It is measure of efforts made by different countries in adopting technologies for reducing carbon footprint
(b) It is measure of environmentally sustainable consumber behaviour in different countries.
(c) It is an assessment of programmes/ schemes undertaken by different countries for improving the conservation of natural resources.
(d) It is an index showing the volume of carbon credits sold by different countries
Ans. b
Explanation:
National Geographic and the international polling firm GlobeScan have just conducted their second annual study measuring and monitoring consumer progress toward environmentally sustainable consumption in 17 countries around the world.
Consumers in India, Brazil and China again scored the highest for green behavior among the countries included the Greendex survey -- and U.S. consumers again scored the worst.

45. Consider the following statement:
1. The Taxus tree naturally found in the Himalayas.
2. The Taxus tree is listed in the Red Data Book.
3. A drug called "taxol" is obtained from Taxus tree is effective against Parkinson's disease.
Which of the statements given above is/are correct?
(a) 1 only
(b) 1 and 2 only
(c) 2 and 3 only
(d) 1, 2 and 3
Ans. b
Explanation:
The Taxus tree naturally found in the Himalayas. The Taxus tree is listed in the Red Data Book. A drug called "taxol" is obtained from Taxus tree is is a mitotic inhibitor used in cancer chemotherapy.

46. P, Q, R and S are four men R is the richest but not the oldest. Q is older than S but not than P or R. P is richer than Q but not than S. The four men can be ordered (descending) in respect of age and richness, respectively, as
(a) PQRS, RPSQ
(b) PRQS, RSPQ
(c) PRQS, RSQP
(d) PRSQ, RSPQ
Ans. b
Explanation:
Consider the following statement-
P is the oldest but not the poorest.
It excludes option c.
Consider the following next statement-
Q is older than S but not than P or R.
It excludes option a and d.

47. What causes wind to deflect toward left in the Southern hemisphere?
(a) Temperature
(b) Magnetic field
(c) Rotation of the earth
(d) Pressure
Ans. c
Explanation:
Moving objects on the surface of the Earth experience a Coriolis force, and appear to veer to the right in the northern hemisphere, and to the left in the southern. The Earth's rotation causes Coriolis force. The Coriolis Effect strongly affects the large-scale oceanic and atmospheric circulation, leading to the formation of robust features like jet streams and western boundary currents.

48. Indiscriminate disposal of used flouorescent electric lamps causes mercury pollution in the environment. Why is mercury used in the manufacture of these lamps?
(a) A mercury coating on the inside of the lamp makes the light bright white
(b) When the lamp is switched on, the mercury in the lamp causes the emission of ultra-violet radiations.
(c) When the lamp is switched on, it is the mercury which converts the ultra-violet energy into visible light
(d) None of the statement given above is correct about the use of mercury in the manufacture of fluorescent lamps
Ans. b
Explanation:
Compact fluorescent bulbs are made of glass tubes filled with gas and a small amount of mercury. CFLs produce light when the mercury molecules are excited by electricity running between two electrodes in the base of the bulb. The mercury emits ultraviolet light, which in turn excites the tube’s phosphor coating, leading it to emit visible light.


49. If there were no Himalayan ranges. Which would have been the most likely geographical impact on India?
1. Much of the country would experience the cold waves from Siberia.
2. Indo-gangetic plain would be devoid of such extensive alluvival soils.
3. The pattern of monsoon would be different from what it is at present.
Which of the statements given above is/are correct?
(a) 1 only
(b) 1 and 3 only
(c) 2 and 3 only
(d) 1, 2 and 3
Ans. d
Explanation:
Himalayan ranges effectively intercept the summer monsoons coming from the Bay of Bengal and Arabian Sea and cause precipitation in the form of rain or snow. They also prevent the cold continental air masses of central Asia from entering into India. The great rivers and their tributaries carry enormous quantities of alluvium. This is deposited in the Great Plain of North India in the form of fertile soils. It is therefore, often said it is Gift of the Himalayas.

50. In the context of space technology, what is "Bhuvan", recently in the news?
(a) A mini satellite launched by ISRO for promoting the distance education in India.
(b) The name given to the next Moon Impact Probe, for Chandrayan-II
(c) A geoportal of ISRO with 3 D imaging capabilities of India.
(d) A space telescope developed by India.
Ans. c
Explanation:
ISRO launched the beta version of its web-based 3-D satellite imagery tool, Bhuvan, on August 12, 2009. Bhuvan will offer superior imagery of Indian locations compared to other Virtual Globe software with spatial resolutions ranging from 10 m to 100 m.

51. The latitudes that pass through Sikkim also pass through
(a) Rajasthan
(b) Punjab
(c) Himachal Pradehs
(d) Jammu & Kashmir
Ans. a
Explanation:
See any Atlas.

52. A man fills a basket with eggs in such a way that the number of eggs added on each successive day is the same as the number already present in the basket. This way the basket gets completely filled in 24 days. After how many days the basket was ¼th full?
(a) 6
(b) 12
(c) 17
(d) 22
Ans. d
Explanation:
Available soon

53. The diameters of two circular coins are in the ratio of 1:3. The smaller coin is made to roll around the bigger coin till it returns to the position from where the process of rolling started. How many times the smaller coin rolled around the bigger coin?
(a) 9
(b) 6
(c) 3
(d) 1.5
Ans. c
Explanation:
Available soon

54. The difference between the interest received from two bank Rs. 500 for two years is Rs. 2.50. is the difference between their rate
(a) 0.25%
(b) 0.5%
(c) 1%
(d) 2.5%
Ans. a
Explanation:
Available soon

55. When ten persons shake hands with another, in how many ways is possible?
(a) 20
(b) 25
(c) 40
(d) 45
Ans. d
Explanation:
Available soon

56. A candidate attempted 12 questions and secured full marks in all of them he obtained 60% in the test and questions carried equal marks, then what is the number of questions in the test
(a) 36
(b) 30
(c) 25
(d) 20
Ans. d
Explanation:
Available soon

57. In how many ways can four children make to stand in a line such that two of them. A and B are always together?
(a) 6
(b) 12
(c) 18
(d) 24
Ans. b
Explanation:
Available soon

58. In a meeting, the map of a village was placed in such a manner that south-east becomes north, north-east becomes west and so on. What south become?
(a) North
(b) North-east
(c) North-west
(d) West
Ans. b
Explanation:
While rotating the map , answer can be fined.

59. Consider the following statements:
1. The "Bombay Manifesto" signed in 1936 openly opposed the preaching of socialist ideals.
2. It evoked support from a large section of business community from all across India.
Which of the statements given above is/are correct?
(a) 1 only
(b) 2 only
(c) Both 1 and 2
(d) Neither 1 nor 2
Ans. c
Explanation:
Available soon

60. Among following, who was not a proponent of bhakti cult?
(a) Nagarjuna
(b) Tukaram
(c) Tyagaraja
(d) Vallabhacharya
Ans. a
Explanation:
Acharya Nāgārjuna was an Indian philosopher who founded the Madhyamaka school of Mahāyāna Buddhism.

61. Due to their extensive rice cultivation, some regions may be contributing to global warming. To what possible reason/reasons is this attributable?
1. The anaerobic conditions associated with rice cultivation cause the emission of methane.
2. When nitrogen based fertilizers are used, nitrous oxide is emitted from the cultivated soil.
Which of the statements given above is/are correct?
(a) 1 only
(b) 2 only
(c) Both 1 and 2
(d) Neither 1 nor 2
Ans. c
Explanation:
Nitrous oxide is produced naturally in soils through the microbial processes of denitrification and nitrification. These natural emissions of N2O can be increased by a variety of agricultural practices and activities, including the use of synthetic and organic fertilizers. The greenhouse gases are carbon dioxide, methane, nitrous oxide , water vapours, carbon particles etc. Growing rice has an adverse environmental impact because of the large quantities of methane gas it generates.

62. Sustaionable development is described as the development that meets the needs of the present without compromising the ability of future generations to meet their own needs. In this perspective, inherently the concept of sustainable development is intertwined with which of the following concepts?
(a) Social justice and empowerment
(b) Inclusive Growth
(c) Globalization
(d) Carrying capacity
Ans. d
Explanation:
The carrying capacity of a biological species in an environment is the population size of the species that the environment can sustain indefinitely, given the food, habitat, water and other necessities available in the environment. Thus Sustaionable development is intertwined with Carrying capacity.

63. Given below are the names of four energy crops. Which one of them can be cultivated for ethanol?
(a) Jatropha
(b) Maize
(c) Pongamia
(d) Sunflower
Ans. b
Explanation:
In India production of alcohol apart from molasses & sugarcane route the non-molasses route like maize, starch, corn grain, sweet sorghum, tapioca, sugar beet are also equally getting importance as the climatic conditions for such type of agricultural crops suits in India.

64. Consider the following pairs:
Protected area Well-known for
1. Bhiterkanika Salt water
Orissa Crocodile
2. Desert Great Indian
National Park Bustard
Rajasthan
3. Eravikulam, Hoolak Gibbon
Kerala
Which of the pairs given above is/are correctly matched
(a) 1 only
(b) 1 and 2 only
(c) 2 only
(d) 1, 2 and 3
Ans. b
Explanation:
A huge population of saltwater crocodiles is present within the Bhitarkanika Wildlife Sanctuary of Orissa and they are known to be present in smaller numbers throughout the Indian and Bangladesh portions of the Sundarbans. Great Indian Bustard is mostly found in the Gujrat, Madhya Pradesh, Jammu and Kashmir, Karnataka, Andhra Pradesh and West Rajasthan. It is mainly spotted in the the Desert National Park (Rajasthan) and the Lala-Parjau sanctuary in western Kutch (Gujarat). Hoolock Gibbons are found in several states of North East India - Assam, Arunachal Pradesh, Manipur, Meghalaya, Tripura and Nagaland.

65. Some species of plants are insectivorous Why?
(a) Their growth in shady and dark places does not allow them to undertake sufficient photosynthesis and theus they depend on insects for nutrition
(b) They are adapted to grow in nitrogen deficient soils and thus depend on insects for sufficient nitrogenous nutrition.
(c) They cannot synthesize certain vitamins themselves and depend on the insects digested by them
(d) They have remained in that particular stage of evolution as living fossils, a link between autotrophs and heterotrophs
Ans.
Explanation:
Insectivorous plants are adapted to grow in nitrogen deficient soils and thus depend on insects for sufficient nitrogenous nutrition.

66. A person travelled a distance of 50 km in 8 hours. He covered a part of the distance on foot at the rate of 4 km per hour and a part on a bicycle at the rate of 10 km per hour. How much distance did he travel on foot?
(a) 10 km
(b) 20 km
(c) 30 km
(d) 40 km
Ans. b
Explanation:
Available soon

67. In the context of governance, consider the following:
1. Encouraging Foreign Direct Investment inflows
2. Privatization of higher educational Institutions
3. Down-sizing of bureaucracy
4. Selling/offoading the shares of Public Sector Undertakings
Which of the above can be used as measures to control the fiscal deficit in India?
(a) 1, 2 and 3
(b) 2, 3 and 4
(c) 1, 2 and 4
(d) 3 and 4 only
Ans. b (Economic Survey 2009-10 page -38)
Explanation:
"To bring the fiscal deficit under control, the government will initiate institutional reform measures which will encompass all aspects of budget such as subsidies, taxes, expenditure and disinvestment".

68. As per the UN-Habitat's Global Report on Human Settlements 2009, which one among the following regions has shown the fastest growth rate of urbanization in the last three decades?
(a) Asia
(b) Europe
(c) Latin America and Caribbean
(d) North America
Ans. a
Explanation:
As per the UN-Habitat's Global Report on Human Settlements 2009 has shown the fastest growth rate of urbanization in the last three decades.

69. In India, which type of forest among the following occupies the largest area?
(a) Montane Wet Temperate Forest
(b) Sub-tropical Dry Evergreen Forest
(c) Tropical Moist Deciduous Forest
(d) Tropical Wet Evergreen Forest
Ans. c
Explanation:
Montane Wet Temperate Forest-3.6%
Sub-tropical Dry Evergreen Forest2.5%
Tropical Moist Deciduous Forest-37%
Tropical Wet Evergreen Forest-8%

70. Inclusive growth as enunciated in the Eleventh Five Year Plan does not include one of the following:
(a) Reduction of poverty
(b) Extension of employment opportunities
(c) Strengthening of capital market
(d) Reduction of gender inequality
Ans. c
Explanation:
Inclusive growth as enunciated in the Eleventh Five Year Plan includes reduction of poverty, extension of employment opportunities and reduction of gender inequality.

71. How many numbers from 0 to 999 are not divisible by either 5 or 7?
(a) 313
(b) 341
(c) 686
(d) 786
Ans. c
Explanation:
Available soon

72. Tamil Nadu is a leading producer of mill-make cotton yarn in the country. What could be the reason?
1. Black cotton soil is the predominant type of soil in the State.
2. Rich pool of skilled labour is available.
Which of the above is/are the correct reasons?
(a) 1 only
(b) 2 only
(c) Both 1 and 2
(d) Neither 1 nor 2
Ans. b
Explanation:
Red soil is the predominant type of soil in the State. Rich pool of skilled labour is main reason for high production of mill-make cotton yarn.

73. Consider the following statements:
1. On the planet Earth, the fresh water available for use amounts to about less than 1% of the total water found.
2. Of the total fresh water found on the planet Earth 95% is bound up in polar ice caps and glaciers.
Which of the statements given above is/are correct?
(a) 1 only
(b) 2 only
(c) Both 1 and 2
(d) Neither 1 nor 2
Ans. a
Explanation:
Only 2.75 percent of the water on Earth is freshwater, about two-thirds of it is frozen in glaciers, a quarter is groundwater and only 0.005 percent of it is surface water.
Less than 1 percent of the world's freshwater is readily accessible.

74. Which one of the following reflects back more sunlight as compared to other three?
(a) Sand desert
(b) Paddy crop land
(c) Land covered with fresh snow
(d) Praire land
Ans. c
Explanation:
The albedo of an object is a measure of how strongly it reflects light from light sources such as the Sun. Snow have a very high albedo and can be as high as 0.9. Because trees tend to have a low albedo, removing forests would tend to increase albedo and thereby could produce localized climate cooling. dry sand have low albedo (0.35) as copare to snow.

75. Rivers that pass through Himachal Pradesh are
(a) Beas and Chenab only
(b) Beas and Ravi only
(c) Chenab, Ravi and Satluj only
(d) Beas, Chenab, Ravi, Satluj and Yamuna
Ans. d
Explanation:
The drainage systems of Himachal Pradesh are the Chandra Bhaga or the Chenab, the Ravi, the Beas, the Sutlej and the Yamuna. These rivers are perennial and are fed by snow and rainfall.

76. Who of the following shall cause every recommendations made by the Finance Commission to be laid before each House of Parliament?
(a) The President of India
(b) The Speaker of Lok Sabha
(c) The Prime Minister of India
(d) The Union Finance Minister
Ans. a
Explanation:
Finnance commission submits its report to the president who lays it before both the House of Parliament along with an explanatory memmorandum as to the action taken on its recommendations.

77. Which one of the following is responsible for the preparation and presentation of Union Budget to the Parliament?
(a) Department of Revenue
(b) Department of Economic Affairs
(c) Department of Financial Services
(d) Department of Expenditure
Ans. b
Explanation:
DEA is the nodal agency of the Union Government to formulate and monitor country's economic policies and programmes having a bearing on domestic and international aspects of economic management. A principal responsibility of this Department is the preparation of the Union Budget annually (excluding the Railway Budget).

78. In a group of five persons A, B, C, D and E, there is a professor, a doctor and lawyer. A and D are unmarried ladies, and o not work. Of the married couple in the Group, E is the husband. B is the brother of A and is neither a doctor nor a lawyer. Who the professor?
(a) B
(b) C
(c) A
(d) Cannot be determined with the available data
Ans. a
Explanation:
Available soon

79. Consider the following actions by the Government:
1. Cutting the tax rates
2. Increasing the government spending
3. Abolishing the subsidies
In the context of economic recession, which of the above actions can be considered a part of the "fiscal stimulus" package?
(a) 1 and 2 only
(b) 2 only
(c) 1 and 3 only
(d) 1, 2 and 3
Ans. a (Economic Survey 2009-10 17)
Explanation:
Steps:
1. Reducing indirect taxes (excise and service tax)
2. Expanding public expenditure
3. Debt relief and subsidies to farmers

80. Half of the villagers of a certain village have their own houses. One-fifth of the villagers cultivate paddy. One-third of the villagers are the literate. Four-fifth of the villagers are below twenty five. Then, which one of the following is certainly true?
(a) All the villagers who have their own houses are literate
(b) Some villagers under twenty five are literate
(c) A quarter of the villagers who have their own houses cultivate paddy
(d) Half of the villagers who cultivate paddy are literate
Ans. b

81. When the Reserve Bank of India announces an increase of the Cash Reserve Ratio, what does it mean?
(a) The commercial banks will have less money to lend
(b) The Reserve Bank of India will have less money to lend
(c) The Union government will have less money to lend
(d) The commercial banks will have more money to lend
Ans. a
Explanation:
This is the amount of money that the banks have to necessarily park with the RBI and reduces lending capacity of commercial banks.

82. Who among the following Governor Generals created the Covenanted Civil Service of India which later came to be known as the Indian Civil Service?
(a) Warren Hastings
(b) Wellesley
(c) Cornwallis
(d) William Bentinck
Ans. c
Explanation:
Cornwallis created the Covenanted Civil Service of India which later came to be known as the Indian Civil Service. A special feature of Covenanted Civil Service of India was the rigid and complete exclusion of Indians from it.

83. What was the immediate cause for the launch of the Swadeshi movement?
(a) The partition of Bengal done by Lord Curzon
(b) A sentence of 18 months rigorous imprisonment imposed on Lokmanya Tilak
(c) The arrest and deportation of Lala Lajpat Rai and Ajit Singh; and passing of the Punjab Colonization Bill
(d) Death sentence pronounced on the Chapekar brothers
Ans. a
Explanation:
Swadeshi Movement emanated from the partition of bengal, 1905 and continued up to 1908. It was the most successful of the pre-Gandhian movements. Chief architects were Aurobindo Ghosh, Veer Savarkar, Lokmanya Bal Gangadhar Tilak, and Lala Lajpat Rai. The Swadeshi Jagaran Manch is an organisation committed to the promotion of Swadeshi (Indigenous) industries and culture.

84. Consider the following statements:
1. Dr. Rajendra Prasad persuaded Mahatma Gandhi to come to Champaran to investigate the problem of peasants.
2. Acharya J.B. Kriplani was one of Mahatma Gandhi's colleagues in his Champaran investigation.
Which of the statements given above is/are correct?
(a) 1 only
(b) 2only
(c) Both 1 and 2
(d) Neither 1 nor 2
Ans. b
Explanation:
Rajkumar Shukla persuaded Mahatma Gandhi to come to Champaran to investigate the problem of peasants. Accompanied by Babu Rajendra Prasad, J. B. Kripalani, Mahadev Desai Gandhijee reached Champaran and began to conduct a detailed inquiry.

85. By a regulation in 1793, the District Collector was deprived of his judicial powers and made the collecting agent only. What was the reason for such regulation?
(a) Lord Cornwallis felt that the District collector's efficiency of revenue collection would enormously increase without the burden of other work
(b) Lord Cornwallis felt that Judicial power should compulsorily be in the hands of Europeans while Indians can be given the job of revenue collection in the districts
(c) Lord Cornwallis was alarmed at the extent of power concentrated in the District Collector and felt that such absolute power was undesirable in one person
(d) The judicial work demanded a deep knowledge of India and a good training in law and Lord Cornwallis felt that District Collector would be only a revenue collector
Ans. c
Explanation:
Lord Cornwallis was alarmed at the extent of power concentrated in the District Collector and felt that such absolute power was undesirable in one person and thus separated the post of the Civil Judge and the Collector.

86. With reference to India, consider the following statements:
1. The Wholesale Price Index (WPI) in India is available on a monthly basis only.
2. As compared to Consumer Price Index for Industrial Workers (CPI(IW), the WPI gives less weight to food articles.
Which of the statements given above is/are correct?
(a) 1 only
(b) 2 only
(c) Both 1 and 2
(d) Neither 1 nor 2
Ans. c
Explanation:
Available soon

87. Each persons' performance compared with all other persons is to be done to rank them subjectively. How many comparisons are needed in total, if there are 11 persons?
(a) 66
(b) 55
(c) 54
(d) 45
Ans. c
Explanation:
Available soon

88. What is the principle by which a cooling system (Radiator) in a motor car works?
(a) Conduction only
(b) Convection
(c) Radiation only
(d) Both conduction and radiation
Ans. b
Explanation:
In practice, the term "radiator" refers to any of a number of devices in which a fluid circulates through exposed pipes notwithstanding that such devices tend to transfer heat mainly by convection and might logically be called convectors.

89. Which among the following do/does not belong/belongs to the GSM family of wireless technologies?
(a) EDGE
(b) LTE
(c) DSL
(d) Both EDGE and LTE
Ans. c
Explanation:
GSM has a straightforward, cost-effective migration path to 3G through GPRS, EDGE and UMTS-HSPA, as well as beyond 3G via the HSPA Evolution (HSPA+), LTE and System Architecture Evolution (SAE) initiatives.

90. With reference to the treatment of cancerous tumours, a tool called cyberknife has been making the news. In this context, which one of the following statement is not correct?
(a) It is robotic image guided system
(b) It delivers an extremely precise dose of radiation
(c) it has the capability of achieving sub-millimetre accuracy
(d) It can map the spread of tumour in the body
Ans. d
Explanation:
The CyberKnife is a frameless robotic radiosurgery system. The CyberKnife system is a method of delivering radiotherapy, with the intention of targeting treatment more accurately than standard radiotherapy.
read more...